patio exam4

अब Quizwiz के साथ अपने होमवर्क और परीक्षाओं को एस करें!

Which medication would the nurse anticipate being prescribed for the client with renal failure who has hyperphosphatemia? A. Vitamin D (Calcitriol) B. Calcium carbonate C. Levothyroxine D. Cinacalcet

swer: B Rationale: Phosphate-binding antacids (aluminum salts, calcium carbonate, or calcium acetate) may be prescribed to decrease the absorption of phosphate from the gastrointestinal tract.

A new client arrives at the clinic. The physician is suspecting the client may have systemic lupus erythematosus (SLE) given the clinical manifestations related to joint pain, skin changes, and history of pleural effusions. The nurse should anticipate which diagnostic test will be a priority to facilitate with the diagnosis? A. Anti-DNA antibody test. B. Routine hemoglobin. C. C-reactive protein. D. B-cell lymphocytes.

Answer: A Rationale: 95% of people with untreated SLE have high ANA levels. However, ANA is not specific for SLE. The anti-DNA antibody test is more specific for the diagnosis of SLE. Hemoglobin may be low if the client has severe anemia but it is not specific for SLE. C-reactive protein will show an inflammatory response but again not specific for SLE.

A 74-year-old woman has been diagnosed with osteoporosis after the bone density scan indicated osteopenia. Which factor would the care team be most likely to rule out as contributors to the client's health problem? A. The woman is black and was exposed to asbestos in her work. B. The client has been a heavy drinker for her whole adult life and has diabetes mellitus. C. The client takes corticosteroids for treatment of her longstanding osteoarthritis. D. The woman has an estrogen deficiency and has never undergone hormone therapy.

Answer: A Rationale: Black race is not a risk factor for the development of osteoporosis, and black perople tend to have higher bone density than white and Hispanic people. Asbestos exposure is not a noted risk factor for osteoporosis, while alcohol use, diabetes, steroids and estrogen-deficiency are all risk factors.

An 80-year-old female with a diagnosis of osteoporosis receives daily supplements of calcitonin in the form of a nasal spray that she instills each morning. Which phenomenon would her care providers expect to result from her supplementation? A. Decreased serum calcium levels B. Increased bone resorption C. Acceleration of osteoclast action D. An increase in bone formation

Answer: A Rationale: Calcitonin lowers both the rate of bone resorption and serum calcium levels. It does not accelerate osteoclast action and it is not noted to increase bone formation.

The nurse is caring for a 22-year-old client with a circumferential burn to the left thigh. What assessment should the nurse perform as a priority specific to this type of burn? A. Color, warmth, circulation, sensation, and movement of the left lower leg and foot B. Swelling or pain in the perineum and genital region and the client's ability to void C. Pain scale and range-of-motion limitations in the left hip compared to the right hip. D. Emotional well-being and any evidence of disturbed body image related to the burn

Answer: A Rationale: Circumferential burns can constrict the involved body part and cause hypoxia and necrosis of the underlying tissues. Such compression of the thigh could result in impaired perfusion to the distal limb, which could be best assessed by observing color, warmth, circulation (capillary refill), sensation, and movement of the left lower leg and foot. Any deficits could indicate the need for an escharotomy to prevent permanent damage to the limb. Unless the perineum is also burned, there should not be any issue with the ability to void. Pain scales, range of motion, and the client's emotional health would be assessed for any burn and are not

Which newborn infant demonstrates the highest risk of presenting with developmental dysplasia of the hip (DDH)? A. A girl who was born with toeing-in and who was in a breech presentation. B. A twin girl who required resuscitation after delivery. C. A boy who was born by cesarian section to a 44-year-old first-time mother. D. A boy with Down syndrome who was exposed to cocaine in utero.

Answer: A Rationale: Female sex, a history of breech presentation, and congenital skeletal abnormalities are all correlated with DDH. Respiratory emergencies, delivery by cesarian section, advanced age of the mother, Down syndrome, and drug exposure are not noted risk factors for DDH.

Following a motorcycle accident which resulted in bilateral femoral fractures, a client has been placed in skeletal traction. The health care providers would recognize which risk is priority to assess for regularly? A. Risk of thromboemboli B. Risk of compartment syndrome C. Risk of permanent muscle atrophy D. Risk of decreased bone density and increased future fracture risk

Answer: A Rationale: For individuals with lower limb injuries, there is a high risk of venous thromboemboli. Compartment syndrome normally manifests within 64 hours of injury and muscle atrophy is not likely to be permanent. His injury and recovery are not likely to result in longstanding decreased bone density.

A client with end stage chronic kidney disease is diagnosed with secondary hyperparathyroidism. Which dietary intervention should the nurse include in the plan of care? A. Restriction of foods high in phosphates B. Encourage increased fluid intake. C. Restriction of foods high in calcium D. Promotion of foods high in vitamin D

Answer: A Rationale: High levels of parathyroid hormone (PTH) will result in increased release of phosphate from bone during mobilization of calcium. In the healthy kidney, PTH stimulates increased excretion of phosphate to help offset the elevation in phosphate. In end-stage kidney disease, the client will not be able to excrete the excess levels of phosphate, leading to hyperphosphatemia. Therefore, the nurse should instruct the client to restrict phosphate-containing foods. Because the phosphate will be elevated, calcium will be low and should not be restricted. Though vitamin D may be promoted, this should not be the priority until phosphate levels are controlled, as elevation of both calcium and phosphate could lead to crystallization in soft tissues. The client with end-stage kidney disease will need to be on a fluid restriction due to the low glomerular filtration rate.

A client arrives with a shoulder dislocation related to sword fighting practice. Which nursing educational topic is most accurate regarding this injury? A. "Dislocations become recurrent. They recur with the same motion but require less and less force each time." B. "If you go to physical therapy as prescribed, you will not likely experience this problem again." C. "Your bones must be weak. You should increase consumption of calcium daily." D. "Let me demonstrate for you how you can put your shoulder back into place if it happens again."

Answer: A Rationale: In the shoulder, dislocation may become recurrent, especially in athletes. They recur with the same motion but require less and less force each time to cause the damage. One should not advocate the client to put the dislocated shoulder back into the socket without medical supervision. Having weak bones refers to osteoporosis, and is not the primary problem for dislocations.

Which client of an oncologist is likely to have the poorest prognosis? A. A 69-year-old man who has been diagnosed with stage IV malignant melanoma. B. A 70-year-old whose skin cancer has been identified as nodular ulcerative basal cell carcinoma. C. A 51-year-old women whose biopsy has revealed intraepidermal squamous cell carcinoma. D. A 59-year-old woman who has invasive squamous cell carcinoma.

Answer: A Rationale: Late detection of malignant melanoma is associated with particularly poor outcomes. Basal cell carcinomas often have strong treatment success rates. While invasive SCC has worse outcomes than intraepidermal SCC, these are both exceeded in mortality by late stage malignant melanoma.

A 20 year-old male has presented to a health clinic with multiple genital lesions that are filled with a viscous, creamy exudate and has subsequently been diagnosed with molluscum contagiosum. What is his care provider most likely to tell him about the plan for treating the diagnosis? A. "This kind of infection often subsides on its own and doesn't grow more serious over time." B. "These lesions usually respond well to oral antiviral medications." C. "Treatment of this problem is usually successful, but if untreated it can lead to sterility." D. "The bacteria that cause this disease are often latent for several years at a time, and you would be contagious for the entire period."

Answer: A Rationale: Molluscum contagiosum is considered benign and self-limiting viral illness. Oral antivirals are not a common treatment and it is not noted to lead to sterility.

Which of these is a clinical manifestation of osteoporosis? A. Loss of mineralized bone mass B. Decreased porosity of bones C. Inability to move joints without pain D. Decrease in joint mobility

Answer: A Rationale: Osteoporosis is a metabolic bone disease characterized by the loss of mineralized bone mass and increased porosity of bones.

The nurse is caring for a client who has been prescribed calcitonin. The current serum calcium level is 6.4 mg/dL (1.6 mmol/L). What is the nurse's best action? A. Withhold the calcitonin until speaking with the health care provider. B. Administer the calcitonin as prescribed and document appropriately. C. Assess client for nausea, constipation, and altered level of consciousness. D. Obtain order for bolus of intravenous fluids and bisphosphonates.

Answer: A Rationale: Prescribed calcitonin aims to lower serum calcium levels. The client's current calcium level is below normal (N=8.5-10.2 mg/dL or 2.13 to 2.55 mmol/L). Therefore, the nurse should withhold the dose until speaking with the health care provider. Nausea, constipation, and altered level of consciousness are symptoms of hypercalcemia, not hypocalcemia. Intravenous fluids and bisphosphonates are treatments for hypercalcemia, as they promote renal elimination of calcium and resorption of calcium in bone.

The infant child of a family who has recently immigrated to the United States from South Asia has been diagnosed with rickets. Bone density scanning would yield which characterization of the infant's bones? A. The child's bones are far softer than those of healthy children. B. The infant has bones that are brittle and susceptible to breakage. C. The child's bones lack bone matrix and prevent weight-bearing. D. The child's bones are oversized due to insufficient osteoclasts.

Answer: A Rationale: Rickets is characterized by soft bones that are undermineralized. Bone matrix is not lacking and the bones tend not to be brittle. Osteoclast deficiency does not underlie rickets.

A public health nurse is planning a vaccination schedule for a local high school. What age group and time frame should the nurse plan for when scheduling the recombinant human papillomavirus vaccine? A. 14-year-old students with a second vaccination in 6 months B. Students who are at least 15 with two more sets of vaccinations 6 months apart C. Students who are sexually active with a second vaccination 1 year later D. 16 year-old students with two additional vaccinations within the next 6 months

Answer: A Rationale: The CDC recommendation for human papillomavirus (HPV) vaccines is two doses given before the adolescent's 15th birthday and spaced out by 6 to 12 months. The nurse could offer the vaccine to older students aged 15 to 26 years in three doses, but the ideal scenario is to vaccinate prior to the 15th birthday (prior to when most students would be sexually active).

A public health nurse has learned that a colleague has been screening for scoliosis during visits made to schools even though such screening is not mandated. How should the nurse best respond to the colleague? A. "The potentially harmful outcomes of screening have been shown to outweigh the benefits." B. "Screening for scoliosis has been proven to be inaccurate." C. "Screening is unnecessary now that we know scoliosis is a benign condition." D. "The low prevalence and incidence of scoliosis have made screening unnecessary."

Answer: A Rationale: The U.S. Preventive Services Task Force recommends against the routine screening of asymptomatic adolescents for idiopathic scoliosis, indicating that the potential harms from screening include unnecessary follow-up visits and evaluations due to false positive results; and psychological adverse effects, especially related to brace wear. Screening is not necessarily inaccurate, and scoliosis is not necessarily benign. The incidence and prevalence remain high.

A client has sustained superficial or first-degree burns to both legs and the left arm. According to the rule of nines, what percentage of total body surface area should the nurse record as affected by this burn? A. 45% B. 36% C. 54% D. 63%

Answer: A Rationale: The rule of nines counts anatomic body parts as multiples of 9% (the head is 9%, each arm 9%, each leg 18%, anterior trunk 18%, posterior trunk 18%), with the perineum 1%. Therefore, the client's burn affect 18% + 18% + 9% = 45% of total body surface area.

Which endocrine functions are responsible for increased resorption of bone that results in a reduction of the number and function of osteoclasts? Select all that apply. A. Calcitonin secretion B. Decrease in estrogen levels C. Stimulation of excess thyroid hormone D. Release of catecholamines E. Increase movement of calcium and phosphate from bone into extracellular fluid.

Answer: A, B Rationale: Parathyroid hormone increases the number of resorptive function of the osteoclasts. Calcitonin is thought to reduce the number and resorptive function of the osteoclasts. Estrogen also reduces the number and function of the osteoclasts. Thus, the decrease in estrogen levels that occurs at menopause results in increased resorption of bone. Release of catecholamines have no influence over osteoclast function. Increased movement of calcium and phosphate from bone into extracellular fluid is regulated by parathyroid hormone.

The public health nurse is tracking the incidence and prevalence of common sexually transmitted infections (STIs) in a population. What factors that influence the statistics related to STIs should the nurse consider when examining the data? Select all that apply. A. Mandatory reporting of cases is only required for three STIs. B. Many STIs are asymptomatic. C. Some common STIs are incurable. D. Clients are often misdiagnosed with STIs. E. Clients can refuse to have their diagnosis reported.

Answer: A, B, C Rationale: The Centers for Disease Control and Prevention only requires reporting of chlamydia, syphilis, and gonorrhea, which leaves many STIs unreported for statistical purposes; this means prevalence would be higher than what can be represented in the data. Also, clients who go undiagnosed due to the asymptomatic nature of some STIs would also not be included in the total. Prevalence is defined as the number of people living with a condition at a given point in time in a population. Therefore, prevalence is elevated by the fact that clients living with viral STIs such as HIV or HSV cannot be cured of the condition. If the condition could be cured, the total number of people with the condition (prevalence) would decrease as they would no longer be counted as living with the condition. Misdiagnoses is not a common issue with STIs that affect statistics. Clients cannot refuse to have their diagnosis reported if it is one requiring mandatory reporting.

The nurse is caring for clients with chronic metabolic disorders. For which of the following clients should the nurse advocate the monitoring of vitamin D levels? Select all that apply. A. A 27-year-old who has had resection of the small bowel B. A 47-year-old with cirrhosis of the liver C. A 60-year-old with advanced chronic kidney disease D. A 55-year-old recovering from knee arthroplasty E. A 15-year-old with viral meningitis

Answer: A, B, C Rationale: Vitamin D must undergo several processes before becoming active. First, it must be absorbed from the small bowel (jejunum), so the client with small bowel resection is at risk for vitamin D deficiency. Also, because vitamin D is fat soluble, the client with cirrhosis who may lack bile salts would be at risk for deficiency. In addition, the liver is involved in the first step of activating vitamin D to form the metabolite 25-hydroxyvitamin D3 [25-(OH)D3]. From the liver, 25-(OH)D3 is transported to the kidneys, where it undergoes conversion to 1,25- dihydroxyvitamin D3 [1,25-(OH)2D3] or 24,25-dihydroxyvitamin D3 [24,25- (OH)2D3] and therefore, vitamin D is often deficient in the person with chronic kidney disease. Neither the client who underwent arthroplasty nor the client with meningitis would be at higher risk for vitamin D deficiency.

The nurse is creating a teaching plan on self-management for a client with moderate rheumatoid arthritis. What should the nurse include in the teaching plan? Select all that apply. A. Use of heat or cold on joints B. Techniques to reduce joint use C. Reduced sodium diet D. Use of assistive devices E. Joining a gym with a personal trainer

Answer: A, B, D Rationale: The nurse should focus the education on the importance of joint support (including the use of splints and assistive devices) and rest balanced with therapeutic exercises. Rather than joining a gym and having the exercise designed by a personal trainer, the client should be advised on therapeutic exercise by a physiotherapist knowledgeable in the management of rheumatoid arthritis. Heat or cold may be used depending on client preference, but there is no need for a reduced sodium diet unless the client is taking medications that cause sodium retention (e.g., corticosteroids).

A health care provider is attempting a differential diagnosis of a 30-year-old female who is suspected of having systemic lupus erythematosus (SLE). Which assessment and history findings correlate with SLE? Select all that apply. A. The client has a "butterfly rash" on her nose and cheeks. B. She complains of intermittent joint pain. C. The woman states that she has numerous environmental allergies. D. The client has been hospitalized twice in the past for pleural effusions. E. Blood work indicates low red cells, white cells, and platelets.

Answer: A, B, D, E Rationale: A butterfly rash, joint pain, pleural effusion and low levels of blood cellular components are all associated with SLE. Environmental allergies are not noted to be risk factors or associated symptoms of the disease.

The nurse is teaching a client about the risk for venous thrombosis and venous thromboemboli as part of discharge instructions. What should the nurse include when teaching the client? Select all that apply. A. The risk for a thromboembolic event lasts for months after you have been discharged. B. If you become suddenly short of breath, call an ambulance immediately. C. Due to the increased risk for embolic stroke and heart attack, carry chewable aspirin. D. Assess the pulse in your affected foot and notify your health care provider if it is pale and cold. E. Notify your health care provider of any evidence of calf swelling and redness.

Answer: A, B, E Rationale: The majority of symptomatic venous thromboemboli associated with hospital admissions occur at least 2 months after hospital discharge. Deep vein thrombi most commonly affect the calf area, so the client should be alert to swelling and redness; because this thrombus could travel to the lungs and become a pulmonary embolus, any sudden respiratory symptoms should prompt the client to seek emergency care. Because arterial blood flow is not affected, there is no need to advise the client to assess for loss of pulse and pallor of the foot. Deep vein thrombi can become a pulmonary embolus but cannot travel to the coronary or cerebral arteries, so there is no increased risk for stroke or heart attack.

A male client arrives at student health services to discuss some "private problems." When alone with the health care provider, he discusses some manifestations of a sexually transmitted infection. Which statements lead the provider to suspect this client has developed genital herpes simplex virus (HSV) type 2? Select all that apply. A. Painful urination B. Cold sore on the lips C. Small 'bumps" on the penile shaft D. Lesions painful to touch. E. Small hemorrhagic areas on penis and scrotum

Answer: A, C, D Rationale: Genital HSV-2 is manifests as pain with an obvious lesion. There can be tingling, itching, and pain in the genital area, followed by eruption of small pustules and vesicles. In men, the infection can cause urethritis (painful urination) and lesions of the penis and scrotum. Trichomoniasis sometimes causes small hemorrhagic areas, usually in females and on the cervix. HSV-1 is usually associated with cold sores but lesions may also appear on the genital area when the person is involved in oral sex.

A client has been admitted with a fractured pelvis that occurred in an auto accident just a few hours ago. The nurse has noticed a slight change in behavior. Which other clinical manifestations would lead the nurse to suspect the client has developed a fat emboli? Select all that apply. A. Reports of substernal chest pain B. Reports of pain in the lower abdomen and back C. Pulse rate 120 with temperature of 99.7°F (37.6°C) D. Profuse diaphoresis with pallor noted E. Urine output of 30 mL/hour

Answer: A, C, D Rationale: Initial symptoms of FES are a subtle change in behavior and signs of disorientation resulting from emboli in the cerebral circulation combined with respiratory depression. There may be reports of substernal chest pain and dyspnea accompanied by tachycardia and low-grade fever. Diaphoresis, pallor, and cyanosis become evident as respiratory function deteriorates. It would be expected that the client has lower abdominal and back pain since he or she has a pelvic fracture. The normal urine output is a minimum of 30 mL/hour.

The nurse is providing education to the child and parents about treatment for the ankle strain the child experienced while playing sports this afternoon. Which topic(s) should the nurse address in this education? Select all that apply. A. Take over-the-counter ibuprofen pain relievers B. Buy a heating pad that has a low-heat setting C. Place an ice pack on the injured area for the next few days D. Pick up a removable splint at the store to provide support to the ankle E. Begin isometric exercise and low-impact walking moves starting tomorrow

Answer: A, C, D Rationale: The treatment of muscle strains include: elevation of the part followed by local application of cold. Compression, using adhesive wraps or removable splint, helps reduce swelling and provides support. Immobiliation for a muscle strain is continued until the pain and swelling have subsided. Warmth is not encouraged until all the swelling is relieved. After a period of immobilization, graded active exercises are started.

The nurse is planning care for a client with hyperparathyroidism. Which interventions should the nurse include in the plan of care? Select all that apply. A. Falls prevention protocol B. Potassium restriction C. Neurologic assessment D. Fluid restriction E. Monitor urine output

Answer: A, C, E Rationale: In hyperparathyroidism, there is an increase in intestinal absorption of calcium and resorption of calcium and phosphate from bone. Bone resorption by the osteoclasts is increased, and bone formation by the osteoblasts is decreased. This causes a weakening of bone and the risk for pathologic fractures, making protection from falls a priority. Parathyroid hormone also promotes the release of phosphate via urine and excess amounts of phosphate in the renal tubules can promote renal calculi formation, making monitoring of urine important. Imbalances in calcium affect neurologic function, so assessment should be conducted. There should not be a restriction of fluids as this would exacerbate hypercalcemia. There would be no indication for potassium restriction.

During assessment of a client with systemic lupus erythematosus (SLE), the nurse hears a friction rub when the stethoscope is placed over the heart. Which complication of SLE will the nurse document in the medical records and report to the health care provider? A. Pleural effusion B. Pericarditis C. Pneumonia D. Vasculitis

Answer: B Rationale: A pericardial friction rub is pathognomonic for acute pericarditis; the rub has a scratching, grating sound similar to leather rubbing against leather. The inner and outer (visceral and parietal, respectively) layers are normally lubricated by a small amount of pericardial fluid, but the inflammation of pericardium causes the walls to rub against each other with audible friction. Pleural effusions are accumulation of fluids within the pleural space and are associated with symptoms of chest pain and shortness of breath. Pneumonia is an infection of the lungs caused by bacteria, viruses, or fungi. General symptoms of pneumonia include chest pain, fever, productive cough, and trouble breathing. Vasculitis is an inflammation of the blood vessels. It happens when the body's immune system attacks the blood vessel by mistake. Vasculitis can affect arteries, veins and capillaries so symptoms depend on the vessel involved.

A client, who has not left the home for many years due to agoraphobia, has agreed to having a home health nurse come for a visit. Knowing this level of isolation, which assessment would be the priority for the nurse to focus on? A. Eye conditions like macular degeneration due to lack of sunlight B. Mobility problems due to lack of vitamin D exposure C. Growths on the throat due to tumors in the parathyroid glands D. Gastrointestinal problems associated with the stress and anxiety

Answer: B Rationale: Agoraphobia often develops after having one or more panic attacks. Agoraphobia is an extreme avoidance of situations that could cause panic. Osteomalacia is the softening of the bones caused by impaired bone metabolism primarily due to inadequate levels of available phosphate, calcium, and vitamin D, or because of resorption of calcium. The impairment of bone metabolism causes inadequate bone mineralization. With isolation indoors, the client likely is not getting any sunlight putting the client at risk for deficiency displayed as osteomalacia. Vitamin D does not primarily cause mobility problems or tumors of the parathyroid gland. Gastrointestinal problems can be related to stress and anxiety but are not primarily caused by being inside all the time. Limited exposure to the sun is not associated with gastrointestinal problems. However, nutritional status would be a priority concern as well if it was listed as a distractor.

A nurse is working on a health care cost analysis related to total knee arthroplasty in the treatment of osteoarthritis. What should the nurse cite as a priority reason to reduce surgery wait times for clients requiring joint replacement? A. The pain clients experience while waiting creates unneeded suffering and loss of productivity. B. Stress placed on joints while awaiting surgery can increase the need for additional surgeries. C. The surgery is less likely to result in complications if performed before joint damage occurs. D. Clients are more likely to seek legal action due to delays, increasing lawsuit costs.

Answer: B Rationale: Although suffering and a decrease in productivity while awaiting surgery are consequences of treatment delays, these are not costs directly incurred by the health care system; accordingly, these are not as relevant to the nurse's focus as is the need for additional surgical procedures. When a client is awaiting surgery on a single knee, the opposite knee often becomes affected because of the additional stress placed on it while trying to protect the involved knee. This can lead to the need for replacement of the second knee. This is more relevant than the risk for litigation. The surgical procedure of total knee arthroplasty would not be indicated for a client if joint damage had not already occurred.

The nurse is changing an occlusive dressing on a stage III pressure ulcer and notes the formation of new eschar in the wound bed. For what change in the wound care plan should the nurse advocate? A. Sending a culture and sensitivity sample. B. Switching to using wet-to-dry dressings. C. Changing the occlusive dressing twice daily. D. Leaving the wound open to air.

Answer: B Rationale: Because necrotic debris increases the possibility of bacterial infection and delays wound healing, this tissue needs to be debrided by using wet-to-dry dressings. Occlusive dressings should not be changed twice daily as this would cause unnecessary disruption to the healing process. This wound should not be left open to air as drying of the tissue will impeded healing. The presence of eschar is not necessarily evidence of a new infection requiring a culture and sensitivity sample.

A nurse is teaching a wellness group among a group of older adult women. One of the women has asked for advice about preventing osteoporosis, which affects many of her friends. What is the nurse's best response to the woman's query? A. "Osteoporosis has been shown to have a strong genetic basis so there is little you can do to prevent it." B. "Weight-bearing exercise is helpful, as are calcium supplements." C. "If possible, scaling back your hormone replacement therapy will reduce your osteoporosis risk." D. "There are drugs called glucocorticoids that both prevent osteoporosis and treat it if you do develop it."

Answer: B Rationale: Exercise and calcium supplementation have been shown to reduce the incidence of osteoporosis. Use of steroids is a risk factor. Estrogen deficiency would contribute to, rather than prevent, osteoporosis. There is a genetic component to the disease but this does not preclude prevention efforts.

The nurse knows that which client listed below is at high risk for developing a hip fracture? A. 77-year-old male who runs marathons and maintains a BMI of 25. B. 82-year-old female with macular degeneration who uses a walker to go to the bathroom. C. 64-year-old male with uncontrolled diabetes and chronic kidney disease on dialysis. D. 73-year-old nursing home client with long-term continence issues but able to walk the hallways for exercise.

Answer: B Rationale: Hip fracture is a major public health problem in the Western world. The incidence of hip fractures increases with age. The incidence is also higher in white women compared with nonwhite women. Risk factors for hip fracture include low BMI, tall body structure, use of benzodiazepines, lack of exercise, previous injury to lower body extremity, vision problems, and confusion.

While reviewing the role of the parathyroid hormone in the balance of calcium and phosphate levels, the nursing faculty will emphasize that the kidney responds to parathyroid stimulation by: A. Increasing reabsorption of sodium in the distal tubules. B. Reducing the reabsorption of phosphate. C. Stimulating production of red blood cells. D. Decreasing the reabsorption of calcium.

Answer: B Rationale: In the kidney, PTH stimulates tubular reabsorption of calcium while reducing the reabsorption of phosphate. The latter effect ensures that increased release of phosphate from bone during mobilization of calcium does not produce an elevation in serum phosphate levels.

An 11-year-old boy with skin lesions on his trunk characteristic of ringworm has been brought to the health care provider. Which aspect of the clinician's assessment relates most directly to the suspected diagnosis? A. Previous infection with other parasitic worms B. Potential contact with the fungus from pets or other children C. Allergic reactions to drugs and environmental substances D. The child's infant immunization history

Answer: B Rationale: Ringworm, or tinea, has a fungal etiology. Parasites, allergies, and immunizations would be unlikely to relate directly to the etiology.

While an infant is in the supine position with both knees flexed, the clinician applies gentle downward pressure to both knees, and the knee and thigh are abducted as an upward and medial pressure is applied to the proximal thigh. This examination technique, used to screen for a reducible dislocation, is called: A. Galeazzi test B. Ortolani maneuver C. Clubfoot test D. Trendelenburg test

Answer: B Rationale: The Ortolani maneuver tests for a reducible dislocation. The Galeazzi test is a measurement of the length of femurs that is done by comparing the height at the knees while they are flexed at 90 degrees. Trendelenburg's test is used for an older child.

A male college student has arrived at the student clinic complaining of tingling, itching, and pains in his groin. Upon inspection, the nurse notices some pustules and vesicles. While taking a detailed sexual history, the nurse should ask which question to rule out further complications? A. "Do you get cold sores very often?" B. "Do you have pain when you urinate or difficulty starting the stream?" C. "Have you noticed excessive swelling in your scrotum the last few days?" D. "Have you been experiencing flank pain and bloody urine?"

Answer: B Rationale: The initial symptoms of primary genital herpes infections include tingling, itching, and pain in the genital area, followed by eruption of small pustules and vesicles. In men, the infection can cause urethritis and lesions of the penis and scrotum. Urethritis is characterized by pain with urination and difficulty starting the stream. Cold sores are not related to genital herpes. Swelling of the scrotum is common in epididymitis. Flank pain and bloody urine are associated primarily with kidney stone formation.

Due to her progressing osteoarthritis (OA), an 80-year-old woman is no longer able to perform her activities of daily living without assistance. Which phenomeon most likely underlies the woman's situation? A. Inappropriate T-cell mediated immune responses have resulted in articular cartilage degeneration. B. Loss of articular cartilage and synovitis has resulted from inflammation caused when joint cartilage attempted to repair itself. C. Excessive collagen deposits have accumulated in the woman's synovial joints. D. Bone overgrowth in synovial joints has resulted in fusing of adjacent bones that normally articulate.

Answer: B Rationale: The joint changes associated with osteoarthritis, which include a progressive loss of articular cartilage and synovitis, results from the inflammation caused when cartilage attempts to repair itself, creating osteophytes or spurs. These changes are accompanied by joint pain, stiffness, limitation of motion, and in some cases by joint instability and deformity. Immune etiology is more associated with rheumatoid arthritis. Collagen deposits are characteristic of scleroderma. Bones do not tend to fuse in the pathogenesis of OA.

A client who has recovered from an initial genital herpes outbreak is seeking advice about controlling the condition. The client asks the nurse, "Can I take antiviral medication now while I don't have sores to prevent another outbreak?" What is the nurse's best response? A. "Yes, you can get a maintenance dose from your physician that is a lower dose compared to the one used during an outbreak." B. "The use of antiviral medication between outbreaks is not recommended unless there is evidence that the virus is active." C. "No, it is not advised as antiviral medication loses its effectiveness over time by causing the virus to mutate and become resistant." D. "This is something decided on a case-by-case basis, so I recommend you speak with your health care provider about your options."

Answer: B Rationale: The nurse should inform the client that during the latent period, the immune system or available treatments have no effect on the virus. Therefore, the reason for not treating the client between outbreaks is unrelated to increased resistance to the treatments. The treatment during the latent phase would not vary between clients. Continuous antiviral suppressive therapy may be advised when more than six outbreaks occur within 1 year (as this would be indicative of viral activity), but since this was the client's initial outbreak, this treatment is not justified.

A 41-year-old woman has been diagnosed as having a loose body of cartilage in her left knee. What data would be most likely to lead clinicians to this diagnosis? A. A visible hematoma is present on the anterior portion of the knee. B. The woman experiences intermittent, painful locking of her joint. C. Computed tomography indicates a complete tear of her knee ligament. D. A X-ray shows that her femoral head and tibia are no longer articulated.

Answer: B Rationale: The symptoms of loose bodies are painful catching and locking of the joint. The loose body repeatedly gets caught in the crevice of a joint, pinching the underlying healthy cartilage. A tear of her ligament, a hematoma, or a lack of articulation are not as closely associated with the presence of loose bodies in a joint.

A 70-year-old woman's family physician has recommended a vitamin D supplement. The woman states that she tries hard to take as few pills as possible and questions her physician on the rationale and necessity of the hormone. How can the physician most accurately reply to the client's concerns? A. "Vitamin D can prevent osteoporosis by increasing the density of your bones." B. "Vitamin D is important in order for your body to absorb the calcium that you consume in your diet." C. "When your liver is unable to produce enough on its own, it's important to take vitamin D supplements to promote bone strength." D. "Vitamin D slows down the rate that your body breaks down your bones."

Answer: B Rationale: Vitamin D facilitates intestinal absorption of calcium. It does not directly increase bone density. The liver does not independently produce vitamin D. Vitamin D does not decrease the rate of bone resorption.

When reviewing the history of clients scheduled for clinical visits today, which client(s) should the provider consider at risk for possible osteopenia? Select all that apply. A. Male, middle-aged, 35 year history of smoking with chronic obstructive pulmonary disease (COPD) B. Female, 40-years old, laboratory value abnormal with high calcium level, low vitamin D level C. Client with a recent history of malignant myeloma currently under treatment D. Perimenopausal 41-year-old woman with history of diabetes currently controlled with insulin E. 30-year-old female whose mother fell last year and had a broken hip

Answer: B, C Rationale: Osteopenia is described as an apparent lack of bone seen on x-ray studies. The major causes of osteopenia are osteoporosis, osteomalacia, malignancies such as multiple myeloma, and endocrine disorders such as hyperparathyroidism and hyperthyroidism. The loss of bone mass is greatest during early menopause, when estrogen levels are withdrawing. Most heath care providers recommend bone density testing for postmenopausal women and men older than 50 years of age according to guidelines.

The nurse is caring for a client who is incontinent and at risk for pressure ulcer formation. What should the nurse include in the plan of care to reduce the risk for pressure ulcer formation? Select all that apply. A. Plan to turn the client when the client reports discomfort. B. Inspect bed sheets for cleanliness each time client is turned. C. Record client's intake and address nutritional deficits. D. Position client in a chair during the day instead of a bed. E. Establish a toileting routine during waking hours

Answer: B, C, E Rationale: Changing the client's position is one of the best ways to prevent pressure ulcers; the position change should not be timed based on reports of discomfort, but on a set schedule of no more than 2 hours. Because moisture from incontinence macerates and injures skin, establishing a toileting routine can reduce skin exposure to this moisture. Food crumbs, intravenous tubing, and other debris in the bed can greatly increase local skin pressure points so the bed sheets should be inspected with each turn. Maintenance of adequate nutrition is important, as malnutrition contributes to tissue breakdown. Placing a client in a chair will increase the risk for ischial decubitus ulcers, and the client would still require repositioning at least every 2 hours rather than being in the chair for the day.

In which client would the nurse expect to find woven bone scaffolding in place to facilitate healing? Select all that apply. A. 14-year-old client recovering from a head injury following a diving accident. B. 22-year-old client recovering from an auto accident where client incurred a fracture of the femur. C. 85-year-old client in a nursing home who is undergoing rehabilitation therapy for the arthritis in their knee. D. 55-year-old client with prostate cancer undergoing radiation therapy for bone metastasis

Answer: B, D Rationale: Woven bone, often referred to as bundle bone, is deposited more rapidly than lamellar bone. It is of low tensile strength, serving as temporary scaffolding for support. It is found in developing fetus, in areas surrounding tumors and infections, and as part of a healing fracture.

While speaking to a senior citizen club about osteoarthritis (OA), which facts are accurate to share? A. By the time people reach their 50s, about 50% of adults will have some form of OA. B. Men usually get OA in their hands whereas women get OA primarily in their hips. C. Obesity in women has been correlated to having OA in the knees. D. Heredity does not play a significant role in the development of OA.

Answer: C Rationale: 85% of people with OA are in their 70s. Men are affected more commonly at a younger age. Heredity influences the occurrence of hand OA in the DIP joint. Hand OA is more likely to affect white women, whereas knee OA is more common in black women. Obesity is a particular risk factor for OA of the knee in women.

An older adult client has arrived at the physician's office complaining of a rash. Upon further investigation, the client states the rash feels like a burning pain but also has some tingling. It is extremely sensitive to touch. The nurse notes that the rash is made up of vesicles and located on the right thoracic region. The nurse suspects the client has: A. Chickenpox B. German measles C. Herpes zoster D. Human papillomavirus

Answer: C Rationale: All of the characteristics point to herpes zoster. The vesicles erupt for 3- 5 days along the nerve pathway (hence the reason for the burning pain). Eruptions usually are unilateral in the thoracic region, trunk, or face. Rubella (German measles) is characterized by a diffuse, punctuate, macular rash that begins on the trunk and spreads to the arms and legs. Varcella (chickenpox) has a macular stage where it develops within hours over the trunk, spreading to the limbs, mucosa, scalp, axillae, upper respiratory tract, and conjunctiva. HPV causes genital warts and is a sexually transmitted infection.

Which changes in an 86-year-old male's skin would necessitate further examination and possible medical treatment? A. A decrease in general sebaceous gland activity. B. Appearance of new skin tags on his chest. C. A new mole-like growth on his forearm. D. Appearance of a keratosis on the man's trunk.

Answer: C Rationale: Appearance of a new mole or a change in the size or shape of an existing mole can be indicative of malignant melanoma. Decreased sebaceous gland secretions, skin tags, and keratoses are normal age-related changes

Which statement reflects an aspect of the bone growth and development that occurs during the first two decades of life? A. Most bone abnormalities can be traced to anomalies in the embryonic stage of development. B. The physiologic effects of in utero positioning normally remain into late adolescence. C. Cartilage cells at the metaphyseal end of the bone plate are replaced by bone cells. D. Bone length increases through childhood while bone diameter remains static.

Answer: C Rationale: During development, the mature and enlarged cartilage cells at the metaphyseal end of the plate become metabolically inactive and are replaced by bone cells. This process allows bone growth to proceed without changing the shape of the bone or causing disruption of the articular cartilage. Abnormalities linked to the embryonic stage of development are uncommon. The physiologic effects of in utero positioning resolve by 3 to 4 years. Both length and diameter of bones increase during development.

While measuring the height of a 12 year-old girl during an office visit, the primary care physician noted a deviation of the girl's spine. This was subsequently quantified as being a right curve scoliosis of approximately 10 degrees deviation. Which of the following treatment options is most clearly indicated? A. Use of a Milwaukee brace during waking hours. B. Surgical correction of the girl's spine. C. Observation and no active treatment. D. Use of external rods to correct the deviation.

Answer: C Rationale: For persons with lesser degrees of curvature (10 to 20 degrees), the trend has been away from aggressive treatment of scoliosis and toward a "wait and see" approach, taking advantage of the more sophisticated diagnostic methods that now are available. The use of surgery or braces is not indicated with a 10 degree curvature.

A public health nurse is conducting a health promotion teaching session at a senior citizens drop-in center. Which teaching point about hip fractures in older adults is most justified? A. "Current treatment options for hip fractures in older adults mean that surgery is no longer a common necessity." B. "Because of their generally higher body mass, men are particularly susceptible to breaking a hip in a fall." C. "Most hip fractures are actually a break at the very top of the thigh bone." D. "Because bone density is largely determined by your genes, there's little you can do to prevent hip fractures other than avoiding falls."

Answer: C Rationale: Fractures to the femoral head are the most common etiology of hip fractures in the older adult. Surgery is commonly required and incidence is higher in women than men. There are numerous risk factors for hip fracture beyond genetic factors, many of which are modifiable.

In which location would a clinician expect to find a ligament? A. At the attachment points between ribs and the sternum. B. Between the radius bone of the arm and the biceps brachii muscle. C. Around the capsule that forms the knee joint. D. Between the individual plates of the skull.

Answer: C Rationale: Ligaments exist where a bone connects to its articulating mate, as in the case of the synovial joint between the tibia and femur that constitutes the knee. Ligaments do not connect the ribs to the sternum. The radius and biceps are connected by a tendon. The interfaces between skull plates are considered joints, but these are not ligaments.

Which client is most likely to have low bone density? A. A 70-year-old woman with increased numbers of osteoblasts. B. A 78-year-old female whose osteoclast function is inhibited. C. A 65-year-old male who is noted to have osteopenia. D. A 68-year-old male who takes vitamin D supplements.

Answer: C Rationale: Osteopenia is characterized by a reduction in bone mass greater than expected for age, race, or sex that occurs because of a decrease in bone formation, inadequate bone mineralization, or excessive bone deossification. Increases in osteoblasts, inhibition of osteoclasts, and vitamin D supplementation would all be associated with high bone density.

. An 18-year-old female of Asian ancestry is distraught over the recent appearance of white patches on her forearms and upper arms which have been subsequently confirmed as vitiligo. Which statement by the woman expresses an accurate understanding of her condition? A. "I suppose it's some comfort that my patches are small and will always stay out of sight in the future." B. "I'm thankful that there are medications to cure this, but I'm nervous about the side effects." C. "I suppose this shouldn't come as too much of surprise, since this tends to run in my family." D. "I'm surprised that I ended up contacting that fungus that caused this problem for me."

Answer: C Rationale: The incidence of vitiligo is thought to have a genetic component. Vitiligo worsens with time and treatments are not curative. The etiology is not infectious and does not involve fungus.

Following a fall 4 weeks prior that was caused by orthostatic hypotension, an 83-year-old male has fractured his femoral head. His care provider has stated that the healing process is occurring at a reasonable pace, and that the man will regain full function after healing and rehabilitation. Which cells are most responsible for restoring the integrity of the man's broken bone? A. Osteocytes B. Osteoclasts C. Osteoblasts D. Osteomas

Answer: C Rationale: The osteoblasts, or bone-building cells, are responsible for the formation of the bone matrix and would participate in the healing process. Osteocytes are mature bone cells. Osteoclasts reabsorb bone cells. An osteoma is a bone tumor.

Following prosthetic joint replacement of the knee, the client continues to complain of pain and discomfort above what would be expected. There is poor incisional healing. The clinic nurse is concerned when she sees that the incision is still draining after 3 weeks post-op. The nurse suspects a prosthetic joint infection. The nurse should anticipate the physician will prescribe: A. complete bed rest with immobilization of the knee. B. high doses of nonsteroidal anti-inflammatory drugs (NSAIDs). C. antimicrobial treatment depending on the microbe. D. wound irrigations with hydrogen peroxide twice daily.

Answer: C Rationale: Treatment includes the use of antibiotics and selective use of surgical interventions. Antimicrobial agents are usually used prophylactically in persons undergoing bone surgery. For persons with osteomyelitis, early antimicrobial treatment—before there is extensive destruction of bone—produces the best results. Best rest is not encouraged. The client may be given NSAIDs for pain but usually prosthetic joint replacement will require a more potent pain killer. Wound irrigations are usually performed in the OR rather than at bedside for infected prosthetic joints.

A client is suspected of having osteomalacia based on clinical manifestations and lab/diagnostic work-up (low calcium levels, transverse lines on X-ray). The nurse would expect the client to have which clinical manifestations? Select all that apply. A. Nerve palsy in upper extremities. B. Calcification of aortic valve. C. Bone pain. D. Muscle weakness. E. Cold limb with absent pulses.

Answer: C, D Rationale: The clinical manifestations of osteomalacia are bone pain, tenderness, and fractures as the disease progresses. In severe cases, muscle weakness often is an early sign. The person is predisposed to pathologic fractures in the weakened areas, especially in the distal radius and proximal femur. Cold limb with absent pulses is a caused by lack of arterial blood supply to the affected bone.

A client comes into the orthopedic clinic complaining of severe pain in the hip that was not caused by a fall. On inspection, the femur and tibia are bowed. There is also a reduced angle of the femoral neck which gives the client a "waddling gait" appearance. The doctor suspects Paget disease, to which the client asks how it was acquired. The nurse will respond: A. "It's because you don't eat enough calcium-rich foods in your diet." B. "When you were a child you probably broke you hip and since it wasn't displaced, you continued to walk on it." C. "It might be related to a thyroid condition. We will need to run some more blood work." D. "It's most likely a genetic predisposition. Do you know if anyone else in your family has this problem?"

Answer: D Rationale: Although the cause of Paget disease remains unclear, there is evidence of both genetic and environmental influences. It has been reported that 15%-40% of people with the disease have a first-degree relative with Paget disease, and numerous studies have described extended family members with the disease.

A 64-year-old man was diagnosed 19 months ago with bilateral osteoarthritis (OA) in his knees, and has come to his family physician for a checkup. The client and his physician are discussing the effects of his health problem and the measures that the man has taken to accommodate and treat his OA in his daily routines. Which statement by the client would necessitate further teaching? A. "I'm really trying to lose weight and I've been able to lose 15 pounds this year so far." B. "I've been doing muscle strengthening exercises twice a week at the community center near my house." C. "Even though I don't like it, I've been using my walker to take some of the weight off my knees." D. "I've been avoiding painkillers because I know they can mask damage that I might be inflicting on my knees."

Answer: D Rationale: Analgesics are a common and appropriate treatment for OA, and it would be unnecessary and inappropriate to forego pain control in order to maximize pain signals from affected joints. Weight loss, the use of assistive devices, and muscle- strengthening exercises are appropriate treatments for OA.

While educating the client starting bisophosphonates for osteoporosis, the client asks, "So what adverse reaction can this drug cause?" Which response by the nurse is most accurate? A. Increase in colon cancer risk B. Development of deep vein thromboses in lower legs C. Adhesion in the esophagus D. A hip fracture

Answer: D Rationale: Bisophosphonates are effective inhibitos of bone resorption and the most effective agents for prevention and treatment of osteoporosis. They have been shown to be effective in reducing the risk of hip, vertebral, and nonvertebral fractures; however, they also have adverse effects such as causing hip fractures although very rare. They are not associated with increase risk of colon cancer, deep vein thromboses or adhesions of the esophagus.

Which teaching point is most appropriate for an adolescent who has sought care for the treatment of severe acne located on the face and back? A. "Avoiding high-fat foods and chocolate won't cure your acne, but it will likely improve it a lot." B. "All the creams and ointments that you can buy have been shown to be no real help for acne." C. "It's important for you to vigorously wash your face several times a day." D. "You might need antibiotic pills in addition to a cream for your face."

Answer: D Rationale: Combination treatments for acne often include a topical preparation in addition to oral antibiotics. Avoiding certain foods has not been shown to be effective in acne treatment or prevention and the topical products available are effective in some, though not all, cases. Care should be taken not to exacerbate lesions by washing the face too vigorously or too often.

While being tackled, a 20 year-old football player put out his hand to break his fall to the ground. Because the intense pain in his wrist did not subside by the end of the game, he was brought to an emergency department, where diagnostic imaging indicated an incomplete tear of the ligament surrounding his wrist joint. At the time of admission, his wrist was swollen with a severely restricted range of motion. What will his care team most likely tell the player about his diagnosis and treatment? A. "This strain will likely resolve itself with sufficient rest." B. "You've suffered a severe sprain and you might need a cast." C. "Your wrist contusion will have to be observed for bleeding under the skin surface." D. "It looks like a mild to moderate sprain and you'll need to keep it immobilized for a few weeks."

Answer: D Rationale: Damage to the ligament structures is associated with sprains, and an incomplete tear would be indicative of a mild to moderate (grade 1 or 2) sprain. A strain is associated most commonly with overuse. A severe sprain would involve total disruption of the ligament.

A nurse working in a long-term care facility has attended an educational conference on pressure ulcer care in older adults. Which protocol in the facility would the nurse advocate changing for the client who develops a stage I ulcer? A. Residents with persistently low food intake are identified. B. Immobilized residents are turned every 2 hours during both day and night. C. Residents are frequently encouraged to increase their fluid intake. D. Wound dressings are applied promptly to all identified or potential pressure ulcers.

Answer: D Rationale: Early stage pressure ulcers and potential pressure ulcers do not necessitate wound dressings and are better treated by turning, keeping skin dry and removing pressure. Nutrition and hydration status are important factors and individuals who cannot reposition themselves independently should be turned regularly.

A 31-year-old male was diagnosed with genital herpes of the HSV-2 type 5 years ago. He is now broaching the subject with a woman he has recently formed a relationship with. Which of his statements is most accurate? A. "If you've been exposed to the herpes virus in the past, then there's no significant risk of reinfection." B. "The worst case scenario is that you'll develop cold sores, since this is the type of herpes virus that I've got." C. "If you've had cold sores when you were younger, it means that you've got antibodies against this type of herpes virus." D. "Even when I'm not having a recurrence, I could still pass the virus on to you."

Answer: D Rationale: HSV transmission can occur both during and between recurrences. HSV- 2 is associated more with genital herpes than cold sores, and cold sores do not provide antibody protection. Prior contact with the virus does not confer immunity.

Which individual who have recently presented to a hospital emergency department is displaying an injury that involves his or her fibrocartilage? A. A 7-year-old girl whose ear has been lacerated during a dog attack. B. A 24-year-old male who has had his trachea crushed in a workplace accident. C. A 15-year-old boy who has suffered a knee injury during a football game. D. A 78-year-old man who has fallen and is suspected of having a "slipped disc" in his back.

Answer: D Rationale: Intervertebral discs are constructed of fibrocartilage, while the ear is largely constructed of elastic cartilage. Cartilage of the respiratory system and joints is most often hyaline cartilage.

A client being investigated for systemic lupus erythematosus (SLE) has an elevated antinuclear antibody result and asks the nurse what this means. What is the nurse's best response? A. This test, combined with your symptoms, is used to determine if you have SLE. B. Antinuclear antibodies are decreased in SLE rather than elevated, so this result is encouraging. C. This test will be repeated in 3 months. If it is still elevated, it supports a diagnosis of SLE. D. This is a nonspecific marker for autoimmune activity, so you will require additional tests.

Answer: D Rationale: Ninety-five percent of people with untreated SLE have high ANA levels, so this result is an indicator of possible SLE. However, because it is not specific for SLE, the nurse should tell the client that additional tests will be required and not state that the ANA test along with the symptoms is sufficient for diagnosis. The client will require an anti-DNA antibody test, which is more specific for the diagnosis of SLE. There is no benefit to waiting and repeating the ANA test instead of testing the anti-DNA.

A nurse is providing care for a client who has had a cast applied to her fractured arm 6 hours prior. The client is now reporting severe pain that she describes as "even worse than when I broke my arm." What would be the nurse's best course of action? A. Administration of analgesics and client teaching about the normal course of pain after a fracture. B. Teaching the client simple range of motion exercises to promote circulation and perfusion. C. Taking the client's temperature due to the possibility of infection. D. Assessment of motor and sensory function with the goal of identifying compartment syndrome.

Answer: D Rationale: Severe pain in the site of a fracture that is out of proportion to the original injury is a hallmark of compartment syndrome. Assessment of sensory and motor function would be an appropriate first action. Analgesia alone and exercises would be insufficient to diagnose or address compartment syndrome. Infection would be an unlikely etiology of sudden pain after cast application.

A hemodynamically stable and conscious client is brought into the emergency department after a crush injury to the pelvis from all-terrain vehicle (ATV) rollover. What action should the nurse prioritize? A. Inserting an intravenous catheter B. Assessing blood pressure and heart rate C. Gaining a detailed history of the incident D. Preventing movement of the pelvic area

Answer: D Rationale: The nurse should be concerned about the risk for fat emboli syndrome (FES) due to pelvic fracture. The only preventive approach to FES is early stabilization of the fracture; therefore, the nurse should ensure that the pelvic area is kept as immobile as possible while performing other actions such as gaining IV access, gathering a history, and assessing vital signs.

Following genetic testing and a thorough history from the child's mother and father, a 5 month-old boy has been diagnosed with osteogenesis imperfecta. What teaching point should the care team provide to the mother and father? A. "His skeleton is prone to breakage and we'll begin hormone therapy to treat this." B. "His hips are extremely susceptible to dislocation, so rough play is out of the question." C. "You'll need to commit to calcium supplementation for the duration of his development." D. "You'll need to be very careful to avoid causing fractures to his fragile bones."

Answer: D Rationale: There is no definitive treatment for correction of the defective collagen synthesis that is characteristic of osteogenesis imperfecta, and prevention and treatment of fractures is important. Hip dislocation is not a common manifestation, and neither hormone therapy nor calcium supplements are useful in treatment.

A 25-year-old woman with primary dysmenorrhea is seeking treatment options from the community health nurse. What should the nurse include in a treatment plan? Select all that apply. A. Determining the cause of the client's symptoms B. Encouraging bed rest during episodes of dysmenorrhea C. Consulting a prescribing provider for SSRI medication D. Consulting a prescribing provider for oral contraceptives E. Administering ibuprofen as needed

Answer: D, E Rationale: Primary dysmenorrhea (pain or discomfort with menstruation) does not have an identifiable cause; this would only apply to secondary dysmenorrhea. Ibuprofen should be recommended as the preferred over-the-counter analgesic rather than others such as acetaminophen. A prescription for oral contraceptives can help alleviate the discomfort. Bed rest is not recommended; the woman can be active as tolerated. She should not be advised to consult about SSRI treatment as this is reserved for severe premenstrual syndrome or premenstrual dysphoric disorders, not for dysmenorrhea.

The nurse is caring for a client in emergent fluid overload requiring rapid diuresis. Which diuretic should the nurse be prepared to administer? A. Loop B. Potassium sparing C. Thiazide D. Osmotic

A nswer: ARationale: Loop diuretics exert their effect in the thick ascending loop of Henle and drugs are the most effective diuretic agents available. They also can be administered intravenously for a rapid diuresis. None of the other classes of diuretic are as potent.

Which phenomenon contributes to the difficulties with absorption, distribution, and elimination of drugs that are associated with kidney disease? A. Reductions in plasma proteins increase the amount of free drug and decrease the amount of protein-bound drug. B. Acute tubular necrosis is associated with impaired drug reabsorption through the tubular epithelium. C. Decreased retention by the kidneys often renders normal drug dosages ineffective. D. Dialysis removes active metabolites from circulation, thereby minimizing therapeutic effect

Answer: A Rationale: A decrease in plasma proteins, particularly albumin, that occurs in many persons with renal failure results in less protein-bound drug and greater amounts of free drug. Drug elimination problems do not stem as directly from impaired tubular reabsorption, decreased retention, or the process of dialysis. Question format: Multiple Choice

To treat enuresis in a young child, the health care provider prescribes desmopressin, an antidiuretic hormone (ADH) nasal spray, before bedtime. What is the most likely rationale for this treatment? A. It removes water from the filtrate and returns it to the vascular compartment. B. It lessens the amount of fluid entering the glomerulus. C. It leads to the production of dilute urine. D. It causes tubular cells to lose their water permeability.

Answer: A Rationale: Antidiuretic hormone (ADH) maintains extracellular volume by returning water to the vascular compartment. This leads to the production of concentrated urine by removing water from the tubular filtrate. In exerting its effect, ADH produces a marked increase in water permeability in tubular cells.

The nurse notes that a client has not voided in over 18 hours and is still unable to void when prompted. What is the nurse's priority assessment? A. Last bowel movement B. Most recent blood glucose level C. Temperature D. Abdominal girth

Answer: A Rationale: Constipation and fecal impaction can result in lower urinary tract obstruction; therefore, this is the priority assessment. Though clients with diabetes can develop neuropathy that can interfere with bladder emptying, this develops over time and is not associated with a specific blood glucose level. The client's temperature is not relevant. Due to the anatomical positioning of the bladder in the pelvis, abdominal girth offers no relevant information.

Of the following list of medications, which would likely be prescribed to a client with benign prostatic hyperplasia (BPH) as a way to decrease the prostate size by blocking the effects of androgens on the prostate? A. Finasteride, a 5-alpha-reductase inhibitor B. Isosorbide mononitrate, a vasodilator C. Birth control pills containing both estrogen and progestin D. Leuprolide, a gonadotropin-releasing hormone analog

Answer: A Rationale: Finasteride, a 5-alpha-reductase inhibitor, reduces prostate size by blocking the effect of androgens on the prostate. Vasodilators, BCPs, and GnRH analogs do not decrease prostate size.

A campus-based peer counseling group is conducting an information blitz on sexually transmitted infections. Which statement about genital warts requires correction? A. "Genital warts can take up to a month after exposure to first become visible." B. "There is no existing treatment that can eradicate the virus once it's contracted." C. "Condoms do not necessarily prevent the transmission of the virus that causes genital warts." D. "There are a number of subtypes of the virus that causes genital warts, but current vaccines protect against most common causes of them."

Answer: A Rationale: Genital warts can take between 6 weeks to 8 months to incubate after exposure. The virus cannot be eliminated and condoms do not provide proven protection. Vaccine protects against 4 HPV types.

The nurse suspects the client may have developed partial bilateral ureter obstruction. What evidence assessment findings support this suspicion? Select all that apply. A. A urine specific gravity of 1.001 B. An increase in urine output C. Low blood pressure D. Weak urine stream E. Bladder distension

Answer: A, B Rationale: The earliest manifestation of partial bilateral obstruction is an inability to concentrate urine, reflected by polyuria and a low specific gravity. Because the obstruction is above the level of the bladder, there will not be urinary retention, bladder distension, or any effect on the strength of urine stream. Hypotension is not expected due to activation of the renin-angiotensin-aldosterone system.

A woman with a diagnosis of type 2 diabetes has been ordered a hemoglobin A1C test for the first time by her primary care provider. The woman states, "I don't see why you want to test my blood cells when it's sugar that's the problem." What aspect of physiology will underlie the care provider's response to the client? A. The amount of glucose attached to A1C cells reflects the average blood glucose levels over the life of the cell. B. Hemoglobin synthesis by the bone marrow is inversely proportionate to blood glucose levels, with low A1C indicating hyperglycemia. C. The high metabolic needs of red cells and their affinity for free glucose indicate the amount of glucose that has been available over 6 to 12 weeks. D. Insulin is a glucose receptor agonist on the hemoglobin molecule and high glucose suggests low insulin levels.

Answer: A Rationale: Glucose entry into red blood cells is not insulin dependent, and the rate at which glucose becomes attached to the hemoglobin molecule depends on blood glucose levels. A1C levels thus indirectly indicate glucose levels. Hemoglobin synthesis, the metabolic needs of hemoglobin, and an agonist role of insulin do not underlie the A1C test.

A client with pancreatitis is admitted with weight loss, nausea, and vomiting. To maintain nutrition, the physician orders parenteral nutrition to be started. Knowing that a major side effect of parenteral nutrition is a hyperosmolar hyperglycemic state, the nurse should assess the client for which clinical manifestations? A. Dry lips, excess urine output, and seizures. B. Facial tics, shuffling gait, stiff joints. C. Fever, chills, elevated BP of 170/101. D. Irritability, bradycardia, wheezing noted on inspiration

Answer: A Rationale: Hyperosmolar hyperglycemic state is characterized by high blood glucose (>600 mg/dL [33.3 mmol/L]), dehydration (dry lips), depression of sensorium, hemiparesis, seizures, and coma. The client may also experience weakness, polyuria, and excessive thirst. HHS may occur in various conditions, including type 2 diabetes, acute pancreatitis, severe infection, MI, and treatment with oral or parenteral nutrition solutions.

A male client is being investigated for infertility and has been told he has a sperm count of 20 million in his sample. He shares with the nurse that he read that you need > 50 million sperm in a sample to be considered fertile. What is the nurse's best response? A. There are several factors that determine fertility and medical advances mean this definition is being updated often. B. Actually the number is 15 million, so you should have no trouble conceiving and do not meet the current definition of infertile. C. Your sperm count is fine, so as long as your motility is also good you will not meet the definition of infertile. D. There are many options for conception even for men whose sperm count meets the definition for infertility.

Answer: A Rationale: Infertility may occur when insufficient numbers of motile, healthy sperm are present. A "fertile sample" on seminal fluid analysis is associated with a count greater than 15 million/mL, greater than 40% motility, but also requires normal morphology, and a volume of 1.5 mL or more so the nurse does not have all the needed information to offer the client reassurance. The field of reproductive endocrinology is greatly expanding and technological advances have meant that the criteria for the definition of infertility is often updated.

The nurse is caring for a client in emergent fluid overload requiring rapid diuresis. Which diuretic should the nurse be prepared to administer? A. Loop B. Potassium sparing C. Thiazide D. Osmotic

Answer: A Rationale: Loop diuretics exert their effect in the thick ascending loop of Henle and drugs are the most effective diuretic agents available. They also can be administered intravenously for a rapid diuresis. None of the other classes of diuretic are as potent.

The nurse is caring for a 34-year-old woman undergoing treatment for large endometrial lesions. What interventions should the nurse anticipate providing the client? A. Providing pain control; preparing the client for a laparoscopic procedure or hysterectomy. B. Administration of packed red blood cells to compensate for low hemoglobin; administering hormone therapy. C. Assisting with a Pap smear; administration of high-dose corticosteroids. D. Administration of male androgens to minimize endometrial hyperplasia; facilitating a dilation and curettage.

Answer: A Rationale: Pain control is central to treatment of endometriosis and surgical treatment is ideally performed laparoscopically, though hysterectomy is sometimes indicated. Bleeding is not a common symptom of endometriosis, and neither corticosteroids nor male androgens are common treatments.

Which statement best captures the essence of a second messenger in the mechanisms of the endocrine system? A. Second messengers act as the intracellular signal that responds to the presence of a hormone. B. Endocrine-producing cells must release both a hormone and a second messenger in order to exert a distant effect. C. Second messengers act to supplement hormone effects on cell receptors when the desired hormonal effect must be either increased. D. Second messengers provide an alternative pathway for endocrine effects on a cell that bypasses the normal receptor pathways.

Answer: A Rationale: Second messengers interact with hormones that cannot cross the cell membrane and they mediate the ultimate effect on the cell. They are not produced by the hormone-producing cell and they are necessary to bring about hormonal effects, not simply for increasing the intensity of the effect. They are not an alternative mechanism of effect but rather a prerequisite for certain hormonal effects on body cells.

Which statement best captures an aspect of the role of hormones in the body? A. Some chemical substances can function as hormones or be integrated with central/peripheral nervous systems. B. Hormones directly initiate many of the processes that contribute to homeostasis. C. Control of body processes is ensured by the fact that a single hormone can only exert one effect on one specific system or tissue. D. Each hormone that exists in the body is produced by only one specific endocrine gland.

Answer: A Rationale: Some chemicals, such as epinephrine, can function as both a hormone and be closely integrated with the central and peripheral nervous systems as well as the immune systems, leading to current terminology such as "neuroendocrine." Hormones modulate, but do not initiate, changes in the body. One hormone may exert multiple effects on multiple body systems. Hormones are produced by a variety of body tissues, not solely by endocrine glands.

Which hormones are derivatives of cholesterol? A. Epinephrine and norepinephrine B. Insulin and glucagon C. Aldosterone and testosterone D. Eicosanoids and retinoids

Answer: C Rationale: Steroids such as aldosterone and testosterone are a classification of hormones that are derived from cholesterol. Epinephrine and norepinephrine are both amino acids. Insulin and glucagon and classified among peptides, polypeptides, proteins, and glycoproteins. Eicosanoids and retinoids consist of fatty acid compounds.

A 40-year-old black client is at increased risk of developing prostate cancer, because his father was diagnosed with the disease at age 60 and his brother at age 56. What diagnostic measures should be undertaken? A. PSA and DRE starting before the age of 50 B. Transrectal ultrasonography starting immediately C. PSA and DRE starting at age 50 D. Transrectal MRI starting at age 45

Answer: A Rationale: The current recommendation of the American Cancer Society and the American Urological Association is that men at increased risk of prostate cancer, such as this client who has two risk factors (his race and his family history), should begin screening with an annual digital rectal exam (DRE) and prostate-specific antigen (PSA) measurement starting at age 45. The general population is advised to undergo this process starting at age 50. It is important to note that PSA levels can be a marker of either benign prostatic hyperplasia or cancer, and there is some degree of controversy regarding the benefit of screening for it. Although transrectal ultrasonography may detect small cancers, its prohibitive cost excludes its routine use in screening.

Which of the following best describes the half-life of a highly protein-bound drug such as thyroxine (99% protein bound)? The half-life would be: A. much longer to reduce the concentration of the hormone by one half. B. shorter because only a little of the hormone has to be used up to reduce the concentration. C. dependent on which drugs were in the blood system holding on to the hormone. D. dependent on the liver to carry the hormone to its designated target organ.

Answer: A Rationale: The half-life of a hormone—the time it takes for the body to reduce the concentration of the hormone by one half—is positively correlated with its percentage of protein binding. Thyroxine, which is more than 99% protein bound, has a half-life of 6 days whereas aldosterone, 15% bound, has a half-life of only 25 minutes.

Which client Is most likely experiencing health problems that may be attributable to kidney disease? A. 81-year-old client with osteoporosis and anemia B. 77-year-old client with urinary retention due to benign prostatic hyperplagia (BPH) C. 55-year-old client with a recent stroke secondary to longstanding hypertension D. 60-year-old client with a systemic fungal infection requiring intravenous antibiotics

Answer: A Rationale: The kidneys play central roles in both vitamin D activation and regulation of red blood cell production, deficiencies of which may result in osteoporosis and anemia, respectively. BPH, stroke, and infection are less likely to result from kidney disease.

An older adult resident of an assisted living facility has been inconsistently continent of urine until the last several weeks. Which intervention by the care providers at the facility is the most likely priority? A. Performing a physical examination and history to determine the exact cause and character of the incontinence. B. Providing client education focusing on the fact that occasional incontinence is a normal, age-related change. C. Teaching the resident about protective pads, collection devices, and medications that may be effective. D. Showing the resident the correct technique for exercises to improve bladder, sphincter, and pelvic floor tone.

Answer: A Rationale: The priority in the treatment of incontinence in older adults is an acknowledgement that it is not an inevitability and that the exact causes should be—and most often can be—identified. This identification by way of history-taking and examination would supersede teaching about protective devices or exercises.

A 3-year-old girl has just been diagnosed with type 1A diabetes. Her parents are currently receiving education from the diabetes education nurse. How can the nurse best explain to the parents the etiology (cause) of their daughter's diabetes? A. "The problem that underlies her diabetes is that her own body has destroyed the cells in her pancreas that produce insulin." B. "It's not known exactly why your daughter has completely stopped making insulin, and treatment will consist of your rigidly controlling her diet." C. "This tendency to produce insufficient amounts of insulin is likely something that she inherited." D. "Environmental and lifestyle factors are known to play a part in the fact that her pancreas secretes and withholds insulin at the wrong times."

Answer: A Rationale: Type 1A, or immune-mediated, diabetes involves the autoimmune destruction of pancreatic beta cells and a consequent absolute lack of insulin. Exogenous insulin is required as dietary control alone is insufficient. The central problem is an absolute lack of insulin production rather than deranged release.

Which substance is most likely to be reabsorbed in the tubular segments of the nephron using passive transport mechanisms? A. Water B. Sodium C. Phosphate D. Calcium

Answer: A Rationale: Water is passively reabsorbed across tubular epithelial membranes while ions like sodium, phosphate, and calcium necessitate active transport

An adult client with a possible growth hormone (GH)-secreting tumor is undergoning testing. If a glucose load is given, which response by the body would confirm the client has a GH-secreting tumor? A. Elevated GH level after a glucose load B. No change in blood level after the glucose load C. Suppresion of the GH level D. Development of gigantism

Answer: A Rationale: When a GH-secreting tumor is suspectred, the GH response to a glucose load is measured as part of the diagnostic workup. Normally, a glucose load would suppress GH levels. However, in adults with GH-secreting tumors (a condition known as acromegaly), GH levels are not suppressed.

A client presents to their health care provider with excruciating back pain progressively getting worse, decreased sensation in his lower extremities, urinary incontinence, and problems completely emptying his bladder. He is diagnosed with advanced prostate cancer with spinal cord compression and admitted to the hospital. The nurse should anticipate administering which medications to this client? Select all that apply. A. Ketoconazole, a fungicide to lower serum testosterone levels B. Bisphosphonates, such as pamidronate, to inhibit bone loss C. Phosphate binding agents such as aluminum hydroxide D. Sulfonamides with trimethoprim to treat the urinary tract infection (UTI) E. Calcium carbonate to prevent osteoporosis

Answer: A, B Rationale: Inhibitors of adrenal androgen synthesis (ketoconazole) are used for treatment of advanced prostate cancer in clients who present with spinal cord compression because these men need rapid decreases in their testosterone levels. The bisphosphonates (pamidronate) act by inhibiting osteoclastic activity. They prevent osteopenia, prevent and delay skeletal complications in clients with metastatic bone involvement, and provide palliation of bone pain. There is no indication that this client has a UTI. Clients with this malignancy usually have hypercalcemia and therefore administration of calcium supplements would be contraindicated.

A nurse is teaching a student about nephrotic syndrome. Which statement(s) about the pathogenesis of nephrotic syndrome is accurate and should be included in the education? Select all that apply. A. Increased glomerular membrane permeability allow proteins to escape from the plasma into glomerular filtrate. B. Massive proteinuria is a result of the increased permeability. C. Loss of colloidal osmotic pressure causes generalized edema. D. The presence of glomerular immunoglobulin A immune complex deposits occur. E. There is an insidious onset beginning with hematuria and mild hypertension

Answer: A, B, C Rationale: Any increase in glomerular membrane permeability allows proteins to escape from the plasma into the glomerular filtrate. Massive proteinuria results, leading to hypoalbuminemia. Generalized edema, which is the hallmark of nephronic syndrome, results from the loss of colloidal osmotic pressure. Membranous glomerulonephritis begins with an insidious onset associated with hematuria and mild hypertension. With Immunoglobulin A nephropathy, the presence of glomerular immunoglobulin A immune complex deposits occur

A nurse is teaching a student about nephrotic syndrome. Which statement(s) about the pathogenesis of nephrotic syndrome is accurate and should be included in the education? Select all that apply. A. Increased glomerular membrane permeability allow proteins to escape from the plasma into glomerular filtrate. B. Massive proteinuria is a result of the increased permeability. C. Loss of colloidal osmotic pressure causes generalized edema. D. The presence of glomerular immunoglobulin A immune complex deposits occur. .E There is an insidious onset beginning with hematuria and mild hypertension

Answer: A, B, C Rationale: Any increase in glomerular membrane permeability allows proteins to escape from the plasma into the glomerular filtrate. Massive proteinuria results, leading to hypoalbuminemia. Generalized edema, which is the hallmark of nephronic syndrome, results from the loss of colloidal osmotic pressure. Membranous glomerulonephritis begins with an insidious onset associated with hematuria and mild hypertension. With Immunoglobulin A nephropathy, the presence of glomerular immunoglobulin A immune complex deposits occur.

The nurse is planning the care for a client with acute kidney injury (AKI). What should the nurse prioritize in the client's plan of care? Select all that apply. A. Assessing fluid balance B. Monitoring electrolyte levels C. Promoting infection control D. Optimizing pain control E. Protecting from falls

Answer: A, B, C Rationale: The nurse will need to monitor fluid balance carefully as the client can experience both fluid volume excess and deficit in AKI. There are also serious consequences due to electrolyte imbalances, such as cardiac dysrhythmias related to hyperkalemia. Secondary infections are a major cause of death in people with AKI, making infection control another priority. Having AKI on its own does not increase the risk for falls or cause pain in the client.

Which clinical manifestations would lead the nurse to suspect that a client with renal failure is developing uremia? Select all that apply. A. Weakness and fatigue. B. Lethargy and confusion. C. Extreme itching. D. Blood in urine. E. Urine smell in the stool

Answer: A, B, C Rationale: Uremia affects all body systems. The symptoms at the onset include weakness, fatigue, nausea, and apathy. These are subtle signs. More severe symptoms include extreme weakness, frequent vomiting, lethargy, and confusion. Pruritis often accompanies the uremic state as well. The term uremia literally means "urine in the blood"; however, it does not cause the appearance of blood in urine (hematuria). There is no direct effect on the gastrointestinal system, so the smell of stool does not change.

A nurse who works in the office of an endocrinologist is orientating a new staff member. Which teaching point is the nurse justified in including in the orientation? Select all that apply. A. "A bodily process can be the result of the combined effect of several different hormones from different sources." B. "A single hormone can act on not only one process or organ, but often on several different locations or processes." C. "It's common for production of hormones to be far removed from the tissue where they ultimately exert their effect." D. "Sometimes hormones act locally on the area where they were produced, like in the case of paracrine and autocrine actions." E. "The regulation in homeostasis requires that hormones be absent from the body when their effect is not needed."

Answer: A, B, C, D Rationale: A single hormone can exert various effects in different tissues or, conversely, a single function can be regulated by several different hormones. Hormones act both distant from their source and more locally, as in the case of autocrine and paracrine actions. Hormones are normally present at all times.

While taking a history on a female client with vague symptoms but who has a strong feeling that "something is wrong with my body," which comments make the health care provider think of ovarian cancer? Select all that apply. A. "For some reason, I have bloating. My pants are tighter than normal." B. "I am losing weight. I just feel full after just a few bits of food." C. "I have to go to the bathroom to urinate frequently. I just cannot hold it very long." D. "I think I must have an infection. I have a heaviness in my groin (pelvic region)." E. "My previous health care provider put me on birth control pills." F. "It seems like all of a sudden, I get diarrhea out of no where."

Answer: A, B, C, D Rationale: Ovarian cancer is often diagnosed at an advanced stage because many symptoms are nonspecific. Symptoms include abdominal bloating or swelling; quickly feeling full when eating; weight loss; discomfort in the pelvis area with pain; change in bowel habits, usually associated with constipation. Home remedies may work for a time, such as antacids, but using them delays diagnosis and treatment. Birth control pills will not affect these symptoms.

A client has suffered damage to the anterior pituitary gland, reducing the ability to respond to increases in plasma osmolality. The nurse should monitor for what expected assessment findings? Select all that apply. A. A urine specific gravity of 1.000 B. A urine to serum osmolality of 1.5:1 C. An increase in body weight D. Increased thirst and fluid consumption E. Decreased urine output

Answer: A, B, D Rationale: Due to damage to the posterior pituitary gland, the client is unable to release antidiuretic hormone (ADH) when there is an increase in serum osmolality. Normally, the body would increase the amount of this hormone to retain water in the renal tubules and dilute the osmolality of the plasma. Lack of ADH means the client will have higher amounts of dilute urine output, which would be reflected by a low specific gravity (N-1.010-1.025). The ratio of urine to serum osmolality would also decrease from the normal ratio of 3:1. There would be weight loss and increased urine output, and also increased thirst and fluid intake in response to the high serum osmolality.

The nurse knows that which clinical manifestations may be present with the diagnosis of acute nephritic syndrome? Select all that apply. A. Sudden onset of hematuria B. Proteinuria C. Flank pain D. Excess urine output E. Edema

Answer: A, B, E Rationale: In its most dramatic form, the acute nephritic syndrome is characterized by sudden onset of hematuria, variable degrees of proteinuria, diminished glomerular filtration rate (GFR), oliguria, and signs of impaired renal function. Inflammatory processes damage the capillary wall. This damage to the capillary wall allows red blood cells to escape into the urine and produce a decrease in GFR. Extracellular fluid accumulation, hypertension, and edema develop because of the decreased GFR. Flank pain is usually associated with kidney stones. Oliguria occurs rather than excess urine output.

A client with hypertension, arthritis, and early chronic kidney disease (CKD) has developed viral gastroenteritis and is unable to consume adequate fluids. Which prescribed medications should the nurse caution the client about taking due to the increased risk for prerenal acute kidney injury? Select all that apply. A. Ramipril B. Ibuprofen C. Acetaminophen D. Amlodipine E. Calcium carbonate F. Hydrochlorothiazide

Answer: A, B, F Rationale: While at risk for dehydration, the client should be cautioned against taking medications that can either worsen dehydration or interfere with renal blood flow autoregulation. Hydrochlorothiazide, a diuretic, will promote dehydration and reduce renal blood flow. Ramipril, an angiotensin-converting enzyme inhibitor, may cause prerenal acute kidney injury in persons with decreased blood flow due to interference with efferent arteriole vasoconstriction. Ibuprofen, an NSAID, can reduce renal blood flow through inhibition of prostaglandin synthesis, which prevents afferent arteriole vasodilation. Acetaminophen is not an NSAID and does not interfere with renal blood flow. Calcium carbonate is an antacid and poses no risk. Amlodipine is a calcium channel blocker that is not linked to pre-renal acute kidney injury, but if the client's blood pressure were low, this medication may need to be avoided while the client is ill.

Following a work-up which included endocrine studies (FSH/LH, prolactin, testosterone, DHEAS levels), a college student complaining of abnormal bleeding has been diagnosed with dysmenorrhea due to alterations in their hormone levels. The nurse should anticipate that which type of medication will be prescribed? Select all that apply. A. Oral contraceptives. B. Estrogen only. C. Prostaglandin synthetase inhibitors. D. Anxiolytic medications. E. Androgens.

Answer: A, C Rationale: The treatment of dysfunctional bleeding of a nonhormonal nature is usually treated with oral contraceptives or cyclic progesterone therapy. Prostaglandin synthetase inhibitors are prescribed for dysmenorrhea. Anxiolytic drugs treat mood changes of PMS.

Which scenario would put a client at risk for experiencing a thyroid problem due to a decrease in thyroxine-binding globulin (TBG)? Select all that apply. A. 55-year-old male with cirrhosis due to alcohol abuse B. 47-year-old female experiencing hot flashes and excess diaphoresis related to menopause C. 75-year-old man receiving chronic glucocorticoid therapy to treat his severe chronic obstructive pulmonary disease (COPD) D. 18-year-old female client with anorexia nervosa weighing 78 pounds who has consumed no protein for the past 3 years.

Answer: A, C, D Rationale: A number of disease conditions and pharmacologic agents can decrease the amount of binding protein in the plasma or influence the binding of hormone. Glucocorticoid medications and systemic disease conditions such as protein malnutrition, nephritic syndrome, and cirrhosis decrease TBG concentrations.

The nurse is providing education to the child and parents about treatment for the ankle strain the child experienced while playing sports this afternoon. Which topic(s) should the nurse address in this education? Select all that apply. A.Take over-the-counter ibuprofen pain relievers B. Buy a heating pad that has a low-heat setting C. Place an ice pack on the injured area for the next few days D. Pick up a removable splint at the store to provide support to the ankle E. Begin isometric exercise and low-impact walking moves starting tomorrow

Answer: A, C, D Rationale: The treatment of muscle strains include: elevation of the part followed by local application of cold. Compression, using adhesive wraps or removable splint, helps reduce swelling and provides support. Immobiliation for a muscle strain is continued until the pain and swelling have subsided. Warmth is not encouraged until all the swelling is relieved. After a period of immobilization, graded active exercises are started. Question format: Multiple Select

A student asks the faculty member, "I just do not get how a hormone can be produced within a neuron and then travel via the blood to affect target cells. Can you give me a couple of examples of this phenonmen?" Which faculty response(s) answers this student's question? Select all that apply. A. Antidiuretic hormone B. Parathyroid hormone C. Testerostone D. Insulin E. Epinephrine

Answer: A, E Rationale: Neuroendocrine actions are when a hormone produced within a neuron then travels through circulation to exert effects on target cells. Vasopressin (antidiuretic hormone) and epinephrine are examples of hormonal substances with neuroendocrine effects. Sex steroids is a paracrine action. Paracrine action or signaling is a form of cell-to-cell communication in which a cell produces a signal to induce changes in nearby cells, altering the behavior of those cells. Insulin is an autocrine hormone. Autocrine action is a mode of hormone action to which hormones bind to receptors on the cell and affects the cell that produces it, like when growth factors that stimulate cell division. Paracrine describes hormone action where hormones are released from cells and bind to receptor on nearby cells and affects their function. The anterior pituitary gland secretes LH which stimulates secretion of testosterone development of interstitial tissue of testes. There is no neuroendocrine action involved in the secretion of testosterone.

Which clinical finding among older adults is most likely to be viewed as a normal part of age-related changes? A. 81-year-old client whose serum creatinine level has increased sharply since the last blood work B. 78-year-old client whose glomerular filtration rate (GFR) has been steadily declining over several years C. 90-year-old client whose blood urea nitrogen (BUN) is rising D. 80-year-old cliet whose dipstick urine reveals protein is present

Answer: B Rationale: A gradual decrease in GFR is considered a normal age-related change. Sudden increase in creatinine or BUN would warrant follow up, as would the presence of protein in a client's urine.

The nurse is caring for a group of four male clients with chronic kidney disease (CKD) and coronary artery disease. The nurse will prioritize which client as being at greatest risk for angina? A. The client with electrocardiogram results indicating sinus bradycardia B. The client with a hemoglobin level of 8.0 g/dL (80 g/L) C. The client with a calcium level of 8 mg/dL (2.0 mmol/L) D. The client with a blood pressure of 160/90 mm Hg

Answer: B Rationale: A hemoglobin level of 8.0 g/dL (80 g/L) is significantly lower than the normal lower limit for males of 14.0 g/dL (140 g/L). This reduction in oxygen- carrying capacity places the client at risk for myocardial ischemia. A mildly lower calcium level does not directly create an imbalance in myocardial oxygen supply and demand. Bradycardia reduces myocardial oxygen demand while increasing supply, compared to tachycardia that can result as a compensatory mechanism in anemia. The elevated blood pressure would not result in angina.

An adult male who is 6'11" tall has a diagnosis of acromegaly. The man is explaining to a curious but sympathetic coworker exactly what accounts for his extraordinary height. Which explanation demonstrates a sound understanding of his health problem? A. "My pituitary gland produced a much higher than normal amount of growth hormone when I was a child." B. "A tumor in my brain threw off my hormone levels after I finished adolescence." C. "My liver is malfunctioning and produces too many of the hormones that ultimately cause growth." D. "The high sugar levels that go along with my diabetes made my pituitary gland overproduce the hormones that cause you to grow."

Answer: B Rationale: Acromegaly is associated with adult onset, and is nearly always involves an adenoma. Increased GH as a child and liver dysfunction are not noted contributors to acromegaly. High levels of GH can cause overproduction of insulin and eventual diabetes, but diabetes does not itself lead to acromegaly.

Which diagnostic bloodwork is most suggestive of chronic kidney disease (CKD)? A. A client with high pH, low levels of calcium, and low levels of phosphate B. A client with low vitamin D levels, low calcitrol levels, and elevated parathyroid hormone (PTH) levels C. A client with low bone density, low levels of calcium, and low levels of phosphate D. A client with low potassium levels, low calcitriol levels, and increased parathyroid hormone (PTH) levels

Answer: B Rationale: CKD is associated with low vitamin D and calcitrol levels, which induces increased PTH production. CKD is also associated with acidosis (low pH), high levels of phosphate and hyperkalemia.

A 20-year-old male has been diagnosed with testicular cancer and is seeking information about his diagnosis from a number of websites. Which statement that he has read is most plausible? A. "Testicular cancer is a leading cause of death among males who should be in the prime of their life." B. "Men with cryptorchidism — the term for an undescended testicle — are known to have a higher risk of developing testicular cancer." C. "For most men with testicular cancer, bloody urine is their first sign that something is wrong." D. "Recent developments in the treatment of testicular cancer mean that few men now need to have a testicle removed."

Answer: B Rationale: Cryptorchidism is an identified risk factor for the development of testicular cancer. The diagnosis is no longer a leading cause of death. Hematuria is not a common symptom and orchiectomy is still the standard treatment modality.

A 60-year-old woman who is 11 years menopausal has presented to the emergency department stating, "I haven't had my period in years, but lately I've been bleeding again, and quite heavily in the last few days." The care team needs to rule out endometrial cancer. How should they best explain the most accurate plan for confirming or ruling out the diagnosis? A. "We're going to book a CT scan for you as soon as possible so that we can see what is inside your uterus." B. "We're going to have to open your cervix with a speculum and take scrapings from the wall of your uterus." C. "We are going to order blood work that will measure your hormone levels." D. "We can do a Pap smear right now and we will get the results as soon as we can."

Answer: B Rationale: D & C is a more accurate diagnostic procedure for endometrial cancer than CT, Pap smear, or blood work analysis.

A client is undergoing an endoscopic resection of the bladder for cancer and the provider has asked the nurse to prepare for intravesical chemotherapy. Which medication will the nurse prepare? A. Lidocaine B. Doxorubicin C. Pembrolizumab D. DTaP

Answer: B Rationale: Doxorubicin is an anthracycline type of chemotherapy used intravesicically for the treatment of bladder cancer. Lidocaine, an anesthetic, is a numbing agent and will not treat the cancer. Pembrolizumab, an immunotherapy agent, is used for the treatment of Hodgkin lymphoma. DTaP is a vaccine that protects against diphtheria and is not a chemotherapeutic. However, the administration of bacille Calmette-Guerin (BCG) vaccine is used as an intervesicular for bladder cancer.

A nurse is collecting a urine specimen prior to measuring the albumin level in a client's urine. A colleague states, "I thought albumin was related to liver function, not the kidney." How can the nurse best respond to this statement? A. "Urine should normally be free of any proteins, and albumin is one of the more common proteins to be excreted in chronic renal failure." B. "Urine albumin levels are useful for diagnosing diabetic kidney disease." C. "A urine dipstick test will tell us exactly how much albumin is being spilled by the kidneys." D. "A urine test for albumin allows us to estimate the glomerular filtration rate quite accurately."

Answer: B Rationale: In clients with diabetes, albumin tests are a useful adjunctive test of nephron injury and repair. Urine is not normally completely free of proteins and a urine dipstick does not allow for the quantification of how much albumin is in a sample. Albumin tests do not allow for an accurate indirect indication of glomerular filtration rate.

During the follicular stage of menstruation, increased estradiol production causes an increase in FSH production. This increase in FSH production by the anterior pituitary gland will have what effect on the follicle? A. Continued growth of the follicle until it can no longer stay in its membrane. B. The follicle will die, which results in a fall of FSH. C. The follicle will continue to grow and produce estradiol. D. The follicle will secrete additional hormones to attract swimming sperm.

Answer: B Rationale: In positive feedback control, rising levels of a hormone cause another gland to release a hormone that is stimulating to the first. Increased estradiol production during the follicular stage of the menstrual cycle causes increased FSH production by the anterior pituitary gland. This stimulates further increases in estradiol levels until the demise of the follicle.

A client with a traumatic amputation of the lower leg has lost >40% of blood volume and is currently not producing any urine output. The nurse bases this phenomena on which humoral substance that is responsible for causing severe vasoconstriction of the renal vessels? A. Aquaporin-2 channels B. Angiotensin II and antidiuretic hormone C. Renin and potassium ions D. Albumin and norepinephrine

Answer: B Rationale: Increased sympathetic activity causes constriction of the afferent arterioles, creating a reduction in renal blood flow. Intense sympathetic stimulation can produce marked decreases in renal blood flow and glomerular filtration rate. Humoral substances, including angiotensin II, antidiuretic hormone, and endothelins produce vasoconstriction of renal blood flow. Aquaporin-2 channels, potassium ions, and albumin do not have vasoconstriction properties.

A nurse practitioner has a 30-year-old male client presenting with fever/chills, urinary frequency/urgency and pain with urination. A urine sample displays cloudy and foul-smelling urine. During digital rectal exam of the prostate, the nurse notes a thick white discharge. A likely diagnosis would be: A. urinary tract infection requiring a prescription of sulfur drugs. B. acute bacterial prostatitis requiring antimicrobial therapy. C. testicular cancer requiring appointment with an urologist. D. inflammation of the epididymis requiring scrotal elevation

Answer: B Rationale: Manifestations of acute bacterial prostatitis include fever and chills, malaise, frequent and urgent urination, and dysuria. The urine may be cloudy and malodorous because of urinary tract infection. Rectal examination reveals a swollen, tender prostate. During exam, prostatic massage produces a thick discharge with WBCs that grows a large numbers of pathogens on culture.

A client with a new diagnosis of an endocrine disorder is unclear how the body can control the levels of different hormones over time. Which statement most accurately underlies the dominant regulation process of hormone levels in the body? A. A positive feedback cycle ensures that stable levels of hormones exist in the body over time. B. With input from various sensors, hormone production and release are adjusted based on existing hormone levels. C. The hypothalamus ensures that hormone levels correspond accurately to the diurnal cycle. D. The pituitary gland is genetically programmed to stimulate and inhibit hormone production and/or release based on needs at different points in the life cycle.

Answer: B Rationale: Most hormone levels are controlled by way of a negative feedback cycle, in which low levels stimulate production and/or release. A positive feedback cycle would not achieve this effect. While some hormones are released on a diurnal schedule, the dominant form of hormone regulation in the body is that of negative feedback. Hormone release is not predetermined by the pituitary gland.

A client has recently been diagnosed with renal calculi and asks the nurse, "What should I be eating to prevent stone formation?" What is the most appropriate response to the client's statement? A. "Your diet may have played a part in this, but in fact genetics are likely primarily to blame." B. "What you eat can influence your risk of stone formation, but many other factors like hormones and your metabolism are involved." C. "You likely do not need to change your diet, but now that you have stones in one kidney, you are at very high risk of growing them in the other kidney." D. "Your diet might be normally healthy, but high intake of normally beneficial minerals like calcium and magnesium can lead to stones."

Answer: B Rationale: Renal calculi have a complex etiology that includes diet, but also many other metabolic and endocrine factors, among others. Genetics are not identified as a contributor and mineral intake is not likely to be the sole factor. Stone formation is normally unilateral.

A medical student is assessing a 22-year-old male who has come to the emergency department because of pain and swelling in his scrotum over the past 36 hours. The attending physician has told the student that she suspects epididymitis. Which of the medical student's following questions is most likely to be useful in the differential diagnosis of epididymitis? A. "Do you know if your vaccinations for mumps are up to date?" B. "Have you had unprotected sex in the past?" C. "Do you have a sensation of heaviness in the left side of your scrotum?" D. "Has it been painful when you get an erection lately?"

Answer: B Rationale: Sexually transmitted infections are a common contributing factor to epididymitis. Mumps are more often a precursor to orchitis, and heaviness on the left side is associated varicocele. Pain with an erection is not a noted symptom of epididymitis.

Following a long history of intermittent back pain and urinary urgency, a 50- year-old client has been diagnosed with chronic bacterial prostatitis. Which factor is most likely to influence his health care provider's choice of treatment? A. The diagnosis is thought to have an autoimmune etiology with limited response to steroid treatments. B. Antibacterial drugs penetrate poorly into the chronically inflamed prostate. C. Urethral catheterization provides symptom relief and contributes to resolution of the underlying infection. D. There are no proven treatments for chronic prostatitis that address the infectious process.

Answer: B Rationale: The fact that antibacterial drugs penetrate poorly into the chronically inflamed prostate makes treatment difficult. Inflammatory prostatitis, not chronic prostatitis, is believed to be an autoimmune problem. Urethral catheterization does not provide resolution of the underlying infection, though treatment modalities do exist.

A 40-year-old man who is morbidly obese and leads a sedentary lifestyle has recently been diagnosed with type 2 diabetes. Which aspects of the man's obesity likely contributed to his new health problem? A. The low metabolic needs of adipose tissue mimic a hypoglycemic state and suppress insulin secretion. B. Free fatty acids contribute to problems such as beta cell dysfunction and insulin resistance. C. Fat tissue initiates glycogenolysis and reliance on glycogen release rather than metabolism of free glucose. D. Triglyceride deposits in the pancreas result in damage to beta cells.

Answer: B Rationale: Type 2 diabetes in obese people is thought to link to the actions of free fatty acids, which include beta cell dysfunction (lipotoxicity), insulin resistance, glucose underutilization, and the accumulation of FFAs and triglycerides to reduce hepatic insulin sensitivity.

An older adult resident of a long term care facility with a recent history of repeated urinary tract infections and restlessness is suspected of having urinary retention. Which intervention by the care team is most appropriate to confirm urine retention? A. Uroflowmetry to determine to rate of the client's urine flow. B. Ultrasound bladder scanning to determine the residual volume of urine after voiding. C. Renal ultrasound aimed at identifying acute or chronic kidney disease. D. Urinalysis focusing on the presence of absence of microorganisms, blood, or white cells in the client's urine

Answer: B Rationale: Ultrasound bladder scanning yields a fast, noninvasive indication of whether or not an individual is adequately emptying his or her bladder with each void. Uroflowmetry would be less indicative of whether the client is retaining urine, and renal ultrasound would address deficits in urine production rather than bladder emptying. Urinalysis would be useful in the diagnosis of infections and/or renal issues more than deficiencies in bladder emptying.

The nurse collects a urine sample but forgets the sample in the client's room for several hours. What is the nurse's best action? A. Place the sample in the fridge immediately. B. Discard the sample and recollect in the morning. C. Label with the time collected and send immediately. D. Contact the laboratory to ask for guidance

Answer: B Rationale: Urine specimens that have been left standing may contain lysed red blood cells, disintegrating casts, and rapidly multiplying bacteria that could cause inaccurate results. The nurse should collect a fresh morning sample and send as soon as possible. This decision does not require input from a laboratory

A nurse is teaching parents of a child who is being discharged after treatment for a streptococcal infection. For what symptoms should the nurse teach the parents to observe, because they are related to the risk for poststreptococcal glomerulonephritis? Select all that apply. A. Pale, dilute urine B. Edema C. Decreased urine output D. Pain in the flank area E. Increased thirst

Answer: B, C Rationale: Poststreptococcal glomerulonephritis can develop 7 to 12 days after a streptococcal infection. The client develops oliguria due to a decrease in glomerular filtration rate. The breakdown of red blood cells present in the urine due to increased capillary permeability results in dark-colored urine rather than pale urine. Sodium and water retention gives rise to edema (particularly of the face and hands) and hypertension. The client would not experience pain or increased thirst with this

When explaining to a client why his prostate is enlarging, the nurse will mention that which hormones may contribute to the prostatic hyperplasia? Select all that apply. A. Glucocorticoids B. Testosterone C. Dihydrotestosterone D. Estrogens E. Progesterone

Answer: B, C, D Rationale: Both androgens (testosterone and dihydrotestosterone) and estrogens appear to contribute to the development of BPH. Testosterone is the most important factor for prostatic growth. DHT, the biologically active metabolite of testosterone, is thought to be the ultimate mediator of prostatic hyperplasia, with estrogen serving to sensitize the prostatic tissue to the growth-producing effects of DHT. Glucocorticoids do not play a role in making the prostate gland enlarge.

A clinic nurse is reviewing laboratory results for several clients with appointments this day. The nurse notes that several of the clients have elevated creatinine levels. Which client(s) would the nurse be most concerned about and highlight the results for the health care provider to review? Select all that apply. A. A client in his early 30s who is training for a triathlon race B. An older adult client whose creatinine livel is 4.5 mg/dL (397.80 μmol/L) C. Adolescent athlete and weight lifter with creatinine level of 2.3 mg/dL (203.32 μmol/L) D. Client with a history of uncontrolled diabetes with creatinine level of 2.7 mg/dL (238.68 μmol/L) E. Client trying to lose weight by increasing the protein level intake by eating meats

Answer: B, D Rationale: Creatinine is a waste product of muscle metabolism. Creatinine levels tend to increase among persons with increased levels of physical activity, higher muscle mass (such as lifting weights), and diets higher in meat. Therefore, the older adult male and the client with uncontrolled diabetes, both with elevated creatinine levels, should be the priority for the health care provider to review and treat accordingly

Which client's laboratory result would be associated with abnormalities in kidney function? Select all that apply. A. An absence of protein in a urine sample B. Increased creatinine levels C. Urine gravity of 1.038 and normal serum creatinine levels D. Decreased blood urea nitrogen level (BUN) E. Detectable levels of glucose in a urine sample F. Elevated cystatin C level

Answer: B, E, F Rationale: Increased creatinine and BUN is associated with abnormalities in renal function, as is the presence of glucose in a urine sample. Urine samples normally lack protein and have a specific gravity of 1.038 to 1.040. An elevated BUN coupled with normal creatinine is likely not attributable to impaired kidney function.

While educating parents about the benefits of giving human papillomavirus recombinant vaccine to their child for prevention of human papillomavirus (HPV), which statements by the parents indicate that the nurse will need to clarify for more accuracy? Select all that apply. A. Best administered before the child becomes sexually active. B. 100% effective against development of cervical cancer for their lifespan. C. Effective against the two most common strains of genital warts. D. Only recommended for females between the ages of 9 and 26. E. Recommended prior to exposure to HPV and if your child has genital warts, he or she is already exposed

Answer: B,D Rationale: The HPV vaccine has decreased the risk of cervical cancer by 97%. HPV 9-valent recombinant vaccine is one type of HPV vaccine to prevent infection with the HPV subtypes 16, 18, 9, and 11. This vaccine has been approved for girls and boys between 9 and 26 years of age (prior to becoming sexually active) to prevent HPV 6 and 11 genital warts. The vaccine targets the two strains of HPV (16, 18) responsible for 70% of cervical cancer. There is no treatment that is 100% effective against cervical cancer.

An example of a single hormone that can exert effects in different tissues is erythropoietin. Erythropoietin is made in the kidney and stimulates the bone marrow to produce: A. Platelets B. Natural killer cells C. Red blood cells D. Mast cells

Answer: C Rationale: A characteristic of hormones is that a single hormone can exert various effects in different tissues. For example, erythropoietin, a traditional circulating hormone, is made in the kidney and stimulates erythropoiesis in the bone marrow. Question format: Multiple Choice Chapter 40: Mechanisms of Endocrine Control

A 51-year-old man is receiving his annual physical exam and his care provider is explaining the rationale for performing a digital rectal exam (DRE). Which statement best captures the rationale for the procedure? A. DRE, combined with a measurement of prostate-specific antigen (PSA), is the easiest way to confirm or rule out benign prostatic hyperplasia (BPH). B. The presence of an enlarged prostate provides a definitive diagnosis of prostate cancer. C. If the prostate is hardened on examination, a biopsy is indicated for further investigation. D. DRE is a screening test recommended for men who are experiencing either reduced urine flow or pain on urination.

Answer: C Rationale: Abnormalities detected during a DRE provide a rationale for further investigation by biopsy. PSA is used to screen for prostate cancer; however, it is also positive in BPH. Therefore, the combination of an elevated PSA and hardened prostate cannot differentiate BPH from prostate cancer. The anatomic location of the prostate at the bladder neck contributes to the pathophysiology and symptomatology of BPH. A BPH is not a definitive sign of prostate cancer. DRE is recommended annually for all men over age 50.

During a thyroid crisis, which medication may increase dissociation of free hormone from the binding proteins, resulting in a worsening effect on the client? A. Lisinipril, an ACE-inhibitor B. Atorvastatin, a statin drug C. Acetylsalicylic acid, a nonsteroidal anti-inflammatory drug (NSAID) D. Glucophage, a biguanide

Answer: C Rationale: Acetylsalicylic acid (aspirin) competes with thyroid hormone for binding to transport proteins. When aspirin is administered to people with excessive levels of circulating thyroid hormone, such as during thyroid crisis, serious effects may occur because of the increased dissociation of free hormone from the binding proteins. The other medications do not have this effect.

Damage to which area of a nephron would most likely result in impaired secretion and reabsorption? A. Distal tubule B. Loop of Henle C. Proximal tubule D. Collecting tubule

Answer: C Rationale: Approximately two-thirds of the absorption and secretion that occurs in the tubular system takes place in the proximal tubule.

A 38-year-old woman takes clomiphene, an infertility drug that works by competing with, and thereby blocking, cellular receptors for estrogen. Which statement is most likely to be true of this client? A. Receptors for all other steroid hormones will also be blocked. B. Up-regulation will increase the number of estrogen receptors on each target cell. C. Estrogen will continue to pass freely through the cellular membranes. D. Laboratory tests will reveal an increase in cyclic adenosine monophosphate (cAMP) levels.

Answer: C Rationale: Because estrogen is a steroid hormone, its receptors in target cells are located inside the cell membrane, and their blockage does not affect the movement of the hormone into and out of the cell. Receptors are specific for each hormone, so no hormones other than estrogen will be blocked. Up-regulation occurs when hormone levels are decreased, and in this case the estrogen level will increase. Second messengers, such as cAMP, are only activated by peptide hormones and catecholamines.

A 15-year-old female has presented to her family physician reporting frequent discomfort around the time of her period. She has subsequently been diagnosed with primary dysmenorrhea. Which treatment is most likely to be effective? A. Investigation and resolution of her hypothalamic-pituitary-ovarian disorder. B. Hormone therapy aimed at resolving her estrogen deprivation. C. Pain control with prostaglandin synthetase inhibitors. D. Dilation and curettage.

Answer: C Rationale: Because she has been diagnosed with primary rather than secondary dysmenorrhea, treatment of a specific underlying condition is not indicated. Treatment will likely focus on adequate pain control. D & C is not a relevant intervention.

A health care provider for an obese male who has a history of diabetes and hypertension reports that the client's glomerular filtration rate (GFR) is 51 mL/min with elevated serum creatinine levels. Which statement by the health care provider will likely answer the client's question about the result? A. "We will regularly monitor your kidney function, but most likely your kidneys will be able to compensate on their own and intervention is not required." B. "You likely have chronic kidney disease and there may be urine in your blood until it is controlled." C. "Your chronic kidney disease has likely been caused by your diabetes and high blood pressure." D. "You are in kidney failure and I will be starting dialysis treatment immediately."

Answer: C Rationale: Diabetes and hypertension are conditions that can cause chronic kidney disease (CKD). While the kidneys do have a remarkable ability to compensate for impaired function, this fact does not mean that treatment would not be undertaken. Hematuria is not a common manifestation of CKD. The client's GFR of 51 mL/min does not indicate kidney failure or the need for dialysis

The nurse administers a glucocorticoid medication to a client with pneumonia. Which of these does the nurse teach the client is the purpose of the medication? A. To regulate glucose metabolism B. To decrease appetite C. To decrease airway inflammation D. To decrease stress of illness

Answer: C Rationale: Glucocorticoids affect metabolism of all nutrients and have anti- inflammatory effects, which can assist with airway inflammation.

During unprotected sex, a 17-year-old female high school senior has been exposed to the human papillomavirus (HPV). The school nurse would recognize that the student is at a considerable risk of developing which diagnosis? A. Genital herpes B. Gonorrhea C. Condylomata acuminata D. Candidiasis

Answer: C Rationale: HPV is an identified precursor to condylomata acuminata, or genital warts. Herpes, gonorrhea, and candidiasis are not likely to result directly from HPV.

A client has been diagnosed with possible IgA nephropathy. The nurse will educate the client about which essential diagnostic test to confirm this diagnosis? A. Urine culture and sensitivity B. Serum blood urea nitrogren (BUN) and creatinine level C. Immunofluorescence microscopy D. Voiding cystourethrogram

Answer: C Rationale: Immunofluorescence microscopy is a powerful technique that is widely used by researchers to assess both the localization and endogenous expression levels of their favorite proteins. It is essential for the diagnosis of IgA nephropathy. Abnormally high levels of creatinine thus warn of possible malfunction or failure of the kidneys. Normal creatinine clearance for healthy women is 88 to 128 mL/min (1.47 to 2.14 mL/s), and 97 to 137 mL/min (1.62 to 2.29 mL/s) for men. Blood urea nitrogen (BUN) level is another indicator of kidney function. Urine culture and sensitivity is used to diagnose a urinary tract infection (UTI) and to identify the bacteria or yeast causing the infection. It may be done in conjunction with susceptibility testing to determine which antibiotics will inhibit the growth of the microbe causing the infection. A voiding cystourethrogram is an x-ray study of the bladder and urethra. It is done while the bladder is emptying.

The mother of 6-year-old male and female fraternal twins has brought her son to see a pediatrician because he is nearly 4 inches shorter than his sister. Which phenomenon would the physician most likely suspect as contributing factor to the boy's short stature? A. Genetic short stature B. Lack of IGF receptors in epiphyseal long bones C. A shortage of hypothalamic GHRH production D. Excess insulin production resulting in chronically low blood glucose levels

Answer: C Rationale: Inadequate levels of hypothalamic GHRH will result in adequate production but inadequate release of GH by the pituitary. Genetic short stature is less likely given the disparity between his height and his twin's, and a shortage of IGF receptors is not a noted pathology. While poorly controlled diabetes can contribute to short stature, excess insulin production is not a likely factor.

A 23-year-old woman has been referred to a fertility clinic after 1 year of attempting to become pregnant. Her diagnostic workup has resulted in a diagnosis of polycystic ovary syndrome (PCOS). What will the first line of treatment most likely consist of? A. Estrogen-progesterone hormone therapy and the administration of clomiphene. B. Surgical resection of the ovaries using laparoscopy. C. Lifestyle modifications to include weight loss by lowering calories and fat consumption. D. Temporary use of oral contractive agents.

Answer: C Rationale: Lifestyle modifications are the treatment of choice for PCOS. Hormone therapy, surgery, and use of oral contraceptives are less likely to be a primary treatment option.

Which teaching point by an oncologist is most appropriate for a 33-year-old male who will begin treatment shortly for his testicular cancer? A. "Thanks to new treatment advances, there's a very good chance that I won't have to remove a testicle." B. "If steroid treatment fails, then we can consider surgery and/or chemotherapy." C. "You should know that there might be adverse effects on your sexual function after treatment is finished." D. "The advantage of performing an orchiectomy over other treatments is that it rules out recurrence."

Answer: C Rationale: Orchiectomy is the standard treatment for testicular cancer. Steroidal treatments are not used and sexual function is commonly affected. Recurrence is possible even after orchiectomy.

A client is in cardiogenic shock following a massive myocardial infarction. The client's family asks the nurse, "Why are the health care providers recommending dialysis since its the heart that is sick?" Which response by the nurse is most appropriate at this time? A. "It looks like your loved one has been exposed to nephrotoxic drugs like a nonsteroidal anti-inflammatory drug (NSAID) prior to the heart attack." B. "When a person has such a large heart attack, the kidneys suffer by developing clots which interfere with urine production." C. "When a person has a large heart attack and goes into shock due to heart failure, there is a decrease in renal perfusion which allows toxins to increase in the blood." D. "It looks like your family member has had a blockage in the ureters for quite some time and the heart attack has made it more difficult for the blood to be filtered by the kidney."

Answer: C Rationale: Prerenal acute kidney injury (AKI) is characterized by a marked decrease in renal blood flow. It is reversible if the cause of the decreased renal blood flow can be identified and corrected before kidney damage occurs. Causes of prerenal AKI include heart failure and cardiogenic shock. This would call for temporary dialysis to filter the blood while the heart is healing. Intrarenal AKI is caused by acute tubular necrosis due to exposure to nephrotoxic drugs or prolonged ischemia. Postrenal AKI is caused by bilateral ureteral obstruction.

To maintain hematocrit levels in clients with kidney failure, the nurse should be prepared to perform which intervention? A. Arrange for frequent blood transfusions in an outpatient clinic. B. Administer iron dextran intravenously. C. Administer a subcutaneous injection of recombinant human erythropoietin (rhEPO). D. Administer prenatal vitamins twice a day.

Answer: C Rationale: Recombinant human erythropoietin (rhEPO) helps maintain hematocrit levels in people with kidney failure. Secondary benefits include improvement in appetite, energy level, sexual function, skin color, hair and nail growth, and reduced cold intolerance. Blood transfusion is a treatment if symptomatic with low hemoglobin; however, the health care provider will try to prevent this by giving EPO. IV iron dextran is for severe iron-deficiency anemia and may be used as part of the treatment of anemia in chonic kidney disease but is not the first choice for iron supplementation. Prenatal vitamins are high in vitamins but not designed for clients with renal failure.

The nurse assesses a client with renal failure for encephalopathy caused by uremia. Which clinical manifestation will the nurse likely find? A. Severe chest pain with pericardial friction rub on auscultation. B. Stiff immobile joints and contractures. C. Loss of recent memory and inattention. D. Pruritus with yellow hue to skin tone

Answer: C Rationale: Reductions in alertness and awareness are the earliest and most significant indications of uremic encephalopathy. These often are followed by an inability to fix attention, loss of recent memory, and perceptual errors in identifying people and objects. Chest pain and friction rub can result from uremia-related pericarditis. Joint pain is not associated with uremia. Although pruritus can occur due to uremia, having a yellow hue to the skin is jaundice (which is associated with liver dysfunction).

A client arrives for "testing to see if my kidney arteries are open and working OK." The nurse interprets this statement and plans to educate the client about which radiologic study? A. A cystoscopy where an instrument is inserted into the bladder to visualize and biopsy specimens. B. An intravenous pyelography exam where dye is injected in a vein and x-ray films are taken as the dye moves through the kidneys. C. Renal angiography where dye is injected directly into the renal artery looking for vascular damage in the renal arteries. D. Computed tomographic scans to outline the kidneys and looking for tumors that may be pressing on the artery.

Answer: C Rationale: Renal angiography involves injecting dye directly into the renal artery to look for vascular damage in the renal arteries or abnormalities of renal blood vessels. Computed tomographic (CT) scans outline the kidneys, look for tumors, and detect renal masses and tumors. An intravenous pyelography exam involves injecting a radiopaque dye into a peripheral vein and taking x-ray films as the dye moves through the kidneys and ureters. With a cystoscopy, an instrument is inserted into the bladder through the urethra to visualize the urethra, bladder, and ureteral insertion sites and to biopsy specimens.

Since steroid hormones are bound to protein carriers for transport, this means: A. they are water-soluble and circulate freely in the blood. B. they are degraded by enzymes in the blood. C. they are inactive in the bound state. D. they will be converted into a useable form by enzymes in the blood.

Answer: C Rationale: Steroid hormones are bound to protein carriers for transport and are inactive in the bound state. Their activity depends on the availability of transport carriers.

A 51-year-old woman has been experiencing signs and symptoms of perimenopause and has sought help from her family physician. A deficiency in estrogen levels has been determined to be a contributing factor. Which phenomenon could potentially underlie the woman's health problem? A. Sufficient synthesis of estrogen but inadequate vesicle-mediated release. B. Inadequate synthesis in the rough endoplasmic reticulum of her ovarian cells. C. Insufficient estrogen production within the smooth endoplasmic reticulum of the relevant cells. D. A lack of prohormone precursors needed for estrogen synthesis and release.

Answer: C Rationale: Steroids such as estrogen are produced in the smooth endoplasmic reticulum. Synthesis and release are not separate processes, as in the case of peptide hormones. Prohormones are associated with peptides, polypeptides, and protein hormones.

A client has visited the health care provider reporting intermittent passing of blood-tinged urine over the last several weeks. Cytology confirms a diagnosis of invasive bladder cancer. Which statement by the provider is most accurate about treatment options? A. "There are new and highly effective chemotherapy regimens that we will investigate." B. "Fortunately bladder cancer has a very low mortality rate, and successful treatment is nearly always possible." C. "It is likely that you will need surgery, possibly a procedure called a cystectomy." D. "Unfortunately there are almost no treatment options for this type of cancer, but we will focus on addressing your symptoms."

Answer: C Rationale: Surgical interventions are common in the treatment of bladder cancer. Effective chemotherapeutic regimens are not yet available, though there are certainly treatment options. The mortality rate of bladder cancer is high, at around 25%.

A nurse is reviewing a client's laboratory results and notices the blood urea nitrogen (BUN):creatinine ratio is 16:1. This ratio most likely correlates to which factor in the client's medical history? A. Hepatitis, a liver disease B. Recent weight loss by following a low-protein diet C. 10-year history of heart failure treated medically D. Chronic hemodialysis three times/week

Answer: C Rationale: The BUN:creatinine ratio provides useful diagnostic information. A normal ratio is approximately 10:1. Ratios higher than 15:1 represent a condition that produces an increase in BUN but not creatinine, such as heart failure or gastrointestinal bleeding. A ratio of less than 10:1 occurs in situations such as liver disease, low-protein diet, or chronic dialysis.

When teaching a class of nursing students, the pathophysiology instructor asks, "What is the majority of energy used for by the kidney?" Which response is most accurate? A. Filtration of drugs out of the body. B. Secretion of erythropoietin for production of red blood cells. C. Active sodium transport mechanisms. D. Removal of excess glucose from the blood.

Answer: C Rationale: The bulk of energy used by the kidney is for active sodium transport mechanisms that facilitate sodium reabsorption and cotransport of other electrolytes and substances such as glucose and amino acids.

A 14-year-old boy has been brought to the emergency department by his mother in excruciating pain that is radiating from his scrotum to his inguinal area. The boy's heart rate is 122 beats per minute and he has vomited twice before arrival at the hospital. Examination reveals that his scrotum is reddened and slightly swollen and the testes are firm to touch and tender, with extensive cremaster muscle contraction noted. What is the boy's most likely diagnosis? A. Epididymitis B. Hydrocele C. Testicular torsion D. Varicocele

Answer: C Rationale: The combination of the boy's age, signs, and symptoms is indicative of testicular torsion. Epididymitis normally lacks cremaster muscle involvement and hydrocele is marked by massive distention of the scrotum. Varicocele is often asymptomatic or marked by heaviness in the scrotum.

A health care provider is caring for an older adult client with a recent diagnosis of renal failure and an acid-base imbalance. The family asks, "What is the underlying cause of the renal failure?" Which phenomenon would most accurately answer the question? A. The kidneys are integral to the reabsorption of hydrogen ions and maintenance of a low pH. B. Blood buffer systems and respiratory control can compensate for inadequate renal control of pH. C. The kidneys have the primary responsibility for eliminating excess hydrogen ions from the body. D. pH is kept at an optimal level through the renal secretion of bicarbonate ions in blood filtrate.

Answer: C Rationale: The kidneys have the primary responsibility for maintaining body pH by eliminating excess hydrogen ions from the body, a function that blood buffer systems and respiratory control are incapable of. Hydrogen ions must be eliminated, not retained, and bicarbonate must be produced as part of buffer action, not secreted.

A 71-year-old man diagnosed with a stage T2 prostate tumor 2 years ago has elected watchful waiting. Recently, his PSA has jumped considerably, as have his levels of serum acid phosphatase. Which course of action would be least appropriate at this stage? A. Combination treatment with an anti-androgen and a GnRH agonist B. Treatment with bisphosphonates C. Treatment with GnRH agonists alone D. Treatment with ketoconazole

Answer: C Rationale: The sharp rise in PSA, coupled with an increase in levels of serum acid phosphatase, is strongly indicative of metastatic cancer (which could be confirmed via molecular imaging such as MRI). Unopposed GnRH agonists initially cause LH and FSH levels to rise, stimulating the production of testosterone, which acts as fuel for the fire of prostatic metastasis. Thus, their use alone would not be appropriate. However, if these agonists are combined with anti-androgens, testosterone levels can be quelled from two different fronts simultaneously. Ketoconazole is a chemical castrating agent that could bring down testosterone levels rapidly and might be more appropriate for cases of advanced and widespread metastasis. Bisphosphonates address the effects of metastatic bone involvement and of osteoporosis resulting from anti-androgen therapy.

A nurse is performing client education with an adult recently diagnosed with chronic kidney disease. Which statement by the client would the nurse most likely want to correct or clarify? A. "I will be prone to anemia, since I am not producing as much of the hormone that causes my bones to produce red blood cells." B. "My heart rate might go up because of my kidney disease and my blood might be a lot thinner than it should be." C. "My kidney problems increase my chance of developing high blood pressure or diabetes." D. "I will have a risk of either bleeding too easily or possibly clotting too quickly, though dialysis can help minimize these effects."

Answer: C Rationale: While high blood pressure can be causative of—or consequent to—renal failure, diabetes is not normally a result of existing chronic kidney disease. Persons with renal failure are indeed prone to anemia, increased heart rate, decreased blood viscosity, and coagulopathies. The risk of bleeding and thrombotic disorders can be partially mitigated by dialysis.

A client with a newly diagnosed adrenal tumor asks the nurse, "So what kind of hormones will be affected if I have my adrenal gland removed?" Which response(s) by the nurse is accurate to relay to this client? Select all that apply. A. "Your growth hormones will be affected." B. "It could interfere with your thyroid hormone levels." C. "You will have changes in your sympathetic nervous system responses to stress." D. "It could affect sodium and potassium electrolyte levels." E. "Blood glucose levels may become elevated."

Answer: C, D, E Rationale: The adrenal cortex is responsible for mineralocorticosteroids, maninly aldosterone which increases sodium absorption and potassium loss by the kidney. Glucocorticoids, mainly cortisol affects regulation of blood glucose levels. Adrenal medulla hormone epinephrine serves as neurtotransmitters for the sympathetic nervous system. Growth hormone which stimulates growth of bone and muscle is secreted by anterior pituitary. The thyroid (follicular cells) increases metabolic rate by releasing T3 and T4.

A client expresses frustration at the inconvenience of having to collect urine for an entire day and night as part of a prescribed 24-hour urine collection test. The client asks the nurse, "Why is this test necessary since I gave a single urine sample 2 days ago?" How could the nurse best respond to this question? A. "A single urine sample lets your care team determine if there are bacteria in your urine, but other tests of urine chemistry need a longer term view." B. "Current lab tests are not able to detect the small quantities of most substances contained in a single urine sample. C. "Only a longer term test is able to show whether your kidneys are letting sugar spill out into your urine." D. "Often when an abnormal substance shows up in urine test, a 24-hour urine collection is needed to determine exactly how much is present in your urine."

Answer: D Rationale: 24-hour urine tests are often used to quantify the amount of substances, such as proteins, that an individual's kidneys are spilling. Single urine samples are able to assess more parameters than just the presence of bacteria, and they are sufficient in quantity to detect numerous substances such as glucose.

A client is admitted to the hospital in Addisonian crisis 1 month after a diagnosis of Addison disease. The nurse knows which clinical manifestation would support this diagnosis? A. Hyperactive deep tendon reflexes and slow, shallow breathing B. Cerebral spinal fluid leakage and impaired swallowing C. Irregular heart rate and decreased temperature D. Change in level of consciousness and profound hypotension

Answer: D Rationale: Acute adrenal crisis is a life-threatening situation. Exposure to even a minor illness or stress can cause a client with Addison disease to develop nausea, vomiting, muscular weakness, hypotension, dehydration, and vascular collapse (which causes a change in LOC). Hemorrhage (low BP) can be caused by septicemia, adrenal trauma, anticoagulant therapy, adrenal vein thrombosis, or adrenal metastases. A hyperactive reflex may indicate disease of the pyramidal tract above the level of the reflex arc being tested. Generalized hyperactivity of DTRs may be caused by hyperthyroidism. Any tear or hole in the membrane that surrounds the brain and spinal cord (dura) can allow the fluid that surrounds those organs to leak. This fluid is called the cerebrospinal fluid (CSF). When it leaks out, the pressure around the brain and spinal cord drops. Causes of leakage through the dura include certain head, brain, or spinal surgeries; head injury; placement of tubes for epidural anesthesia or pain medications; or lumbar puncture. Irregular heart rates (dysrhythmias) may be caused by many different factors, including coronary artery disease, electrolyte imbalances in the blood (such as sodium or potassium), changes in the heart muscle, or injury from a heart attack.

A young adult client has just been diagnosed with acute glomerulonephritis. Which question should the nurse ask this client in attempting to establish a cause? A. "Do you have a history of heart failure?" B. "Have you recently had kidney stones?" C. "Have you ever been diagnosed with diabetes?" D. "Have you had any type of infection within the past 2 weeks?"

Answer: D Rationale: Acute postinfectious glomerulonephritis usually occurs after infection with certain strains of group A beta-hemolytic streptococci and is caused by deposition of immune complexes of antibody and bacterial antigens. Other organism can also cause this infection

A client asks the nurse, "The doctor said I have adenocarcinoma of the bladder. What does this mean?" What information should the nurse provide to the client related to the pathophysiologic principles of this type of cancer? A. It is a low-grade tumor that is readily cured with chemotherapy. B. After resection of the tumor, the prognosis is excellent with this type of cancer. C. This cancer invades local tissue, therefore the entire bladder must be removed. D. This is a rare and highly metastatic form of bladder cancer

Answer: D Rationale: Adenocarcinoma is rare and highly metastatic. Due to the likelihood of metastases, the nurse should not indicate that resection, chemotherapy, or cystectomy alone will be curative.

An endocrinologist is providing care for a 30-year-old male who has lived with the effects of increased levels of growth hormone (GH). Which teaching point about the client's future health risks is most accurate? A. "It's not unusual for high GH levels to cause damage to your hypothalamus." B. "GH excess inhibits your pancreas from producing enough insulin." C. "The high levels of GH that circulate in your body can result in damage to your liver." D. "When your pituitary gland is enlarged, there's a real risk that you'll develop some sight deficiencies."

Answer: D Rationale: GH excess is associated with tumor formation and consequent compression of cranial nerves is responsible for vision. Damage to the hypothalamus and liver are not common sequelae. While the beta cells of the pancreas can "burn out," the primary effect of excess GH is to increase insulin secretion.

When explaining about structural classifications to a group of students, the instructor discusses the peptides and proteins. The instructor talks about small hormones and hormones as large and complex as growth hormone (GH), which has approximately how many amino acids involved? A. 50 amino acids B. 100 amino acids C. 150 amino acids D. 200 amino acids

Answer: D Rationale: Growth hormone is a very large and complex protein which has approximately 200 amino acids.

An adult diagnosed with renal failure secondary to diabetes mellitus is scheduled to begin dialysis soon. Which statement by the client reflects an accurate understanding of the process of hemodialysis? A. "It is stressful knowing that committing to dialysis means I cannot qualify for a kidney transplant." B. "I know I will have to go to a hospital or dialysis center for treatment." C. "Changing my schedule to accommodate 3 or 4 hours of hemodialysis each day will be difficult." D. "I will not be able to go about my normal routine during treatment."

Answer: D Rationale: Hemodialysis requires the client to remain connected to dialysis machinery, whereas peritoneal dialysis allows for activity during treatment. Dialysis does not disqualify an individual from receiving a transplant. Dialysis does not require attendance at a dialysis center; clients can be taught to perform the dialysis in their home with a family member in attendance. Hemodialysis is normally conducted 3 times weekly, not once per day.

An older adult male resident has a grossly distended scrotum. On examination, the resident has been diagnosed with a hydrocele. What will the nurse most likely tell the resident and his family about his diagnosis? A. "We'll get you to the hospital quickly, because if this isn't treated it can result in a blockage in the blood flow to your testes." B. "This isn't a result of your intestine entering your scrotum, but an accumulation of fluid within your testes." C. "This can sometimes result from the lower level of sex hormones that comes with age, so you will likely benefit from a testosterone supplement." D. "While distressing to look at, this condition usually doesn't have any significant consequences and won't need treatment."

Answer: D Rationale: Hydrocele in adult males is normally considered a benign condition that does not warrant treatment unless mobility is affected. Fluid accumulates in the space between the tunica vaginalis and tunica albuginea, not within the testes themselves, and hormone therapy is not indicated.

Which statement best captures the relationship between the hypothalamus and the pituitary gland as it relates to endocrine function? A. The hypothalamus directly measures the levels of most hormones throughout the body and inhibits or stimulates the pituitary accordingly. B. The pituitary gland coordinates and dictates the release of hormones from the hypothalamus that act on their intended target cells. C. The pituitary gland and hypothalamus have two-way communication that mediates the signals from neuronal inputs. D. The hypothalamus receives input from numerous sources throughout the body and directs the pituitary to then control many target glands and cells.

Answer: D Rationale: The hypothalamus can be viewed as a bridge by which signals from multiple systems are relayed to the pituitary gland. The hypothalamus collects data from sources throughout the body rather than directly measuring levels. Communication normally flows from the hypothalamus to the pituitary.

Which statement most accurately captures the function of the ascending loop of Henle? A. Urine is concentrated by the selective absorption of free water in the ascending limb. B. Sodium and water are reabsorbing in equal amounts, reducing filtrate quantity but maintaining osmolality. C. The majority of solute and water reabsorption occur in the ascending loop of Henle. D. Impermeability to water and absorption of solutes yields highly dilute filtrate.

Answer: D Rationale: The osmolality of filtrate reaches a low of 100 mOsm/kg (100 mmol/kg) of water in the ascending limb as a result of its impermeability to water.

Which statement best captures an aspect of normal spermatogenesis? A. Testosterone chemically lyses each primary spermatocyte into two secondary spermatocytes with 23 chromosomes each. B. Sertoli cells differentiate into spermatids, each of which can contribute half of the chromosomes necessary for reproduction. C. Spermatogonia adjacent to the tubular wall undergo meiotic division and provide a continuous source of new germinal cells. D. Each primary spermatocyte undergoes two nuclear divisions yielding four cells with 23 chromosomes each.

Answer: D Rationale: The process of meiosis consists of two consecutive nuclear divisions of a primary spermatocyte with formation of four daughter cells, each containing a single set of 23 chromosomes. Testosterone does not chemically lyse immature sperm. Sertoli cells play a nurturing and facilitative role in spermatogenesis rather than differentiating themselves into spermatocytes. Spermatogonia undergo mitosis, not meiosis.

During the study of hormones related to sexual functioning, the instructor mentions negative feedback systems. Which is an example pertaining to a negative feedback loop in the male genitourinary stystem? A. High levels of testerone suppress the secretion of sperm. B. Follicle stimulating hormone (FSH) stimulates inhibin and inhibin supresses FSH. C. Lack of anterior pituitary secretion due to low levels of follicle stimulating hormone (FSH). D. Low androgens and sperm counts due to high levels of gonadotropins.

B Rationale: The negative feedback actions of terstosterone regulate circulating levels of the gonadotropic hormones: high levels of testostrone suppress the secretions of luteinizing hormone through a direct action on the pituitary. Inhibin, produced by Sertoli cells, inhibit FSH. Inhibin suppresses FSH release from the pituitary gland. A classic example of a negative feedback loop is when FSH stimulates inhibin and inhibin supresseses FSH.


संबंधित स्टडी सेट्स

Children and Adolescents Coursepoint Questions

View Set

Lecture 5 Transport Layer Security

View Set

AP Gov Chapter 19 Study Guide Questions

View Set